MRCP Deck 2 May 2023 Flashcards

1
Q

A 24M frequently engages in unprotected sex with multiple partners. He had a mildly raised Venereal Disease Research Laboratory (VDRL) test at 1:8. He did however have a negative EIA and TPPA test. You suspect it could be a false positive test result. Which of the following would be useful at determining a cause?
A) HIV test
B) RF
C) Electrophoresis
D) Varicella serology
E) Mycoplasma serology

A

The VDRL test is very sensitive for syphilis infections and titres can be used to track treatment and progression. It is, however, prone to many false positives (a positive VDRL in the absence of a positive EIA/TPPA). False positives are usually due to a reaction of antibodies to the cardiolipin-lecithin-cholesterol reagent in the RPR/VDRL tests. SLE, HIV, antiphospholipid syndrome and TB infection are classic causes of this.

False positive VDRL/RPR: ‘SomeTimes Mistakes Happen’ (SLE, TB, malaria, HIV)

Example of test results:
-Positive non-treponemal test + positive treponemal test = active syphilis infection
-Positive non-treponemal test + negative treponemal test = consistent with a false-positive syphilis result
-Negative non-treponemal test + positive treponemal test = successfully treated syphilis

How well did you know this?
1
Not at all
2
3
4
5
Perfectly
2
Q

A 19M undergoes primary immunisation against hepatitis B. Following the full course of vaccines, his post immunisation bloods show Anti-HBs< 10 mIU/ml. What is the most appropriate course of action?
A) Give one further dose of Hep B
B) HIV test
C) Test for current or past Hep B infection and repeat full course
D) Give 2 further doses of Hep B vaccine
E) Give a course of HBIg and one further dose of Hep B vaccine

A

An antibody level below 10mIU/ml is classified as a non-response to vaccine, and testing for markers of current or past infection is good clinical practice. In non-responders, a repeat course of vaccine is recommended, followed by retesting one to two months after the second course. Those who still have anti-HBs levels below 10mIU/ml, and who have no markers of current or past infection, will require HBIG for protection if exposed to the virus.

How well did you know this?
1
Not at all
2
3
4
5
Perfectly
3
Q

A 55M with a history of IHD presents with palpitations for the past 10 days. He has a pulse of 130 bpm which is irregularly irregular. He has had one previous episode of atrial fibrillation 3 months ago which was terminated by elective cardioversion following warfarinisation. What term best describes his arrhythmia?
A) Persistent AF
B) Atrial flutter
C) Permanent AF
D) Paroxysmal AF
E) Secondary AF

A

-First detected episode
-Recurrent episodes = 2 or more episodes of AF. If episodes of AF terminate spontaneously then the term paroxysmal AF is used. Such episodes last less than 7 days (typically < 24 hours). If the arrhythmia is not self-terminating then the term persistent AF is used. Such episodes usually last greater than 7 days
-Permanent AF = continuous atrial fibrillation which cannot be cardioverted. Treatment goals are therefore rate control and anticoagulation if appropriate.

How well did you know this?
1
Not at all
2
3
4
5
Perfectly
4
Q

Which of the following may cause a decompensation in chronic liver disease?
A) High fibre diet
B) Low protein diet
C) Constipation
D) Diarrhoea
E) High carbohydrate diet

A

Constipation is a common cause of liver decompensation due to the accumulation of toxic products within the body. Some patients with liver decompensation and hepatic encephalopathy are treated with enemas to reduce the uptake of toxic products.

Other common causes include infection, electrolyte imbalances, dehydration, upper GI bleeds or increased alcohol intake.

How well did you know this?
1
Not at all
2
3
4
5
Perfectly
5
Q

What is the best treatment for a patient presenting in haemolytic crisis secondary to hereditary spherocytosis?
A) Eculizumab
B) Folic acid
C) IVIG
D) Splenectomy
E) Steroids

A

Patients with HS have chronic low-grade haemolysis and therefore an increased red cell turnover. Since folic acid is essential for haematopoiesis, all patients with HS should continue on lifelong folic acid replacement. This is even more essential in a haemolytic crisis. Importantly this remains the case even when folic acid levels are normal.

Indications for splenectomy in HS include severe symptomatic anaemia, transfusion dependence, or gallstone disease.

How well did you know this?
1
Not at all
2
3
4
5
Perfectly
6
Q

Which of the following is characteristic of atrial myxoma?
A) It originates from the R atrium
B) Fragments of tumour break off and metastasise
C) Echo is diagnostic in most cases
D) The clinical signs can mimic MR
E) Recurrence is frequent, even after surgical removal

A

Atrial myxomas are benign tumours of the heart that are more common in men. They consist of a triad of: embolism, intracardiac obstruction and constitutional symptoms. 2D echo is usually sufficient to make the diagnosis.

It usually originates from the L atrium and clinical signs mimic MS rather than MR.

How well did you know this?
1
Not at all
2
3
4
5
Perfectly
7
Q

Which of the options is a specific requirement for MDR-TB compared to other variants of TB?
A) A combination of at least 4 drugs are recommended
B) Directly observed therapy is recommended
C) There is usually no requirement for fluroquinolones
D) Treatment is required for at least 6M
E) Treatment failure is defined as 2 positive blood cultures after 2 months of therapy

A

It is difficult to confirm adequate adherence to therapy, and eradication of TB is essential. As such, as much as possible, DOT should be instigated.

How well did you know this?
1
Not at all
2
3
4
5
Perfectly
8
Q

Which of the following is expected to be seen on a biopsy of a granuloma in sarcoidosis?
A) Asteroid bodies
B) B lymphocytes
C) T lymphocytes
D) Caseation
E) Heinz bodies

A

Asteroid bodies are found in sarcoid granulomas and are thought to comprise lipids arranged into bilayer membranes.

How well did you know this?
1
Not at all
2
3
4
5
Perfectly
9
Q

Which of the following medications is most likely to precipitate hepatic encephalopathy in a patient with advanced alcoholic liver disease?
A) Amlodipine
B) Diazepam
C) Folic acid
D) Thiamine
E) Vitamin E

A

Patients with liver disease are more susceptible to the effects of various sedative medications on the CNS: benzodiazepines, opioids, barbiturates and chlorpromazine.

How well did you know this?
1
Not at all
2
3
4
5
Perfectly
10
Q

Which of the following is the most appropriate definitive treatment for a hyperthyroid solitary toxic nodule?
A) Radioactive iodine therapy
B) Carbimazole
C) Surgical excision
D) Propranolol
E) High dose carbimazole therapy with thyroxine therapy

A

Toxic thyroid nodules are particularly amenable to treatment with radioactive iodine treatment and if the nodule is shrinking 3M post treatment then no further assessment is required. Surgical excision is used second line or in women who are lactating or are pregnant (radioactive iodine is not appropriate).

How well did you know this?
1
Not at all
2
3
4
5
Perfectly
11
Q

What are two common complications of seborrhoeic dermatitis?
A) Alopecia and otitis externa
B) Blepharitis and otitis externa
C) Photosensitivity and alopecia
D) Photosensitivity and blepharitis
E) Blepharitis and alopecia

A

Otitis externa and blepharitis are common complications of seborrhoeic dermatitis.

How well did you know this?
1
Not at all
2
3
4
5
Perfectly
12
Q

A 14-year-old boy is admitted with palpitations and is noted to have a long QT interval. His only past medical history is deafness. What is the likely diagnosis?
A) Leriche syndrome
B) WPW syndrome
C) Jerville-Lange-Nielson syndrome
D) Romano-Ward syndrome
E) Osler-Wender-Rendu syndrome

A

Jervell-Lange-Nielsen syndrome is associated with profound deafness and a prolonged QT interval.

Romano-Ward syndrome is also associated with a prolonged QT interval but no deafness.

How well did you know this?
1
Not at all
2
3
4
5
Perfectly
13
Q

Which feature would be the strongest indication for referral of a hyperparathyroid patient for consideration of parathyroid surgery?
A) Co-existing T2DM
B) Post-menopausal patient
C) Persistent hypercalcaemia over 4Y
D) Vitamin D deficiency
E) 45 years of age

A

Treatment of hyperparathyroidism:
-The definitive management is total parathyroidectomy
-Conservative management may be offered if the calcium level is less than 0.25 mmol/L above the upper limit of normal AND the patient is > 50 years AND there is no evidence of end-organ damage
-Patients not suitable for surgery may be treated with cinacalcet (‘mimics’ the action of calcium on tissues by allosteric activation of the calcium-sensing receptor)

NICE guidelines clearly stipulate the circumstances under which parathyroidectomy should be considered in primary hyperparathyroidism. These are listed below:

-<50 years.
-Cca that is >0.25 mmol/L above the upper end of the reference range.
-eGFR <60 mL/min/1.73 m2 a
-Renal stones
-Osteoporosis/osteoporotic fracture.
-Symptomatic disease

How well did you know this?
1
Not at all
2
3
4
5
Perfectly
14
Q

What follow up is required for a patient found to have pleural plaques on CT with no abnormalities noted on their blood tests?
A) None
B) Palliative chemotherapy
C) Radiotherapy
D) Surgery
E) Repeat CT in 6 months

A

Pleural plaques are benign and do not undergo malignant change. They, therefore don’t require any follow-up.

How well did you know this?
1
Not at all
2
3
4
5
Perfectly
15
Q

On what chromosome is the most likely mutation of a patient with ADPKD?
A) 3
B) 16
C) 4
D) 12
E) 6

A

The majority of patients with ADPKD have a mutation on chromosome 16, with the remaining 15% having a mutation on chromosome 4.

The screening investigation for relatives is abdominal ultrasound:
-2 cysts, unilateral or bilateral, if < 30YO
-2 cysts in both kidneys if 30-59 YO
-4 cysts in both kidneys if > 60 YO

Tolvaptan MAY be used to treat depending on if the patient meets the criteria.

How well did you know this?
1
Not at all
2
3
4
5
Perfectly
16
Q

What is the mechanism of action of amphotericin B?
A) Inhibits DNA polymerase
B) Converted to 5-fluoruracil
C) Binds with ergosterol
D) Inhibits synthesis of B-glucan
E) Interacts with microtubules to disrupt the mitotic spindle

A

Amphotericin B binds with ergosterol, a component of fungal cell membranes, forming pores that cause lysis of the cell wall and subsequent fungal cell death

How well did you know this?
1
Not at all
2
3
4
5
Perfectly
17
Q

A 65M presents with a sudden, painless loss of vision in his right eye upon waking up this morning. PMHx is notable for DM and HTN. Fundoscopic examination of the left eye reveals no abnormalities but of the right eye reveals multiple retinal haemorrhages. What is the most likely diagnosis?
A) Amaurosis fugax
B) Central retinal artery occlusion
C) Central retinal vein occlusion
D) Hypertensive retinopathy
E) Optic neuritis

A

Central retinal vein occlusion - sudden painless loss of vision, severe retinal haemorrhages on fundoscopy

-Amaurosis fugax = temporary vision loss that resolves in a few minutes. It is caused by atheroembolism from the carotid arteries.
-Central retinal artery occlusion = acute painless vision loss, loss of relative afferent pupillary defect, and on fundoscopy, a cherry-red macula and a white/pale retina.
-Optic neuritis = sudden loss of vision associated with severe pain and an afferent pupillary defect.

How well did you know this?
1
Not at all
2
3
4
5
Perfectly
18
Q

In terms of the cell cycle, which one of the following phases determine the length of the cell cycle:
A) M
B) M0
C) G1
D) S
E) G2

A

G1 is where the cells increase in size and determines the length of the cell cycle.

How well did you know this?
1
Not at all
2
3
4
5
Perfectly
19
Q

A patient wakes up an hour ago with right-sided weakness and difficulty speaking. A CT head does not show any signs of intracranial haemorrhage and a CT angiogram reveals a clot in the first branch (M1) of the left middle cerebral artery (MCA). Which of the following interventions is most likely to be of benefit in this patient?
A) Thrombolysis
B) Mechanical clot retrieval
C) Treatment dose heparin
D) Mechanical clot retrieval and thrombolysis
E) DAPT

A

Thrombolysis is contra-indicated in patients with wake-up strokes but mechanical clot retrieval can be performed in patients who have a confirmed occlusion of the proximal anterior circulation.

Contraindications to thrombolysis: >4.5hrs since symptoms due to risk of haemorrhagic transformation, presence of an intracranial neoplasm, recent major bleeding e.g. from the gastrointestinal tract.

How well did you know this?
1
Not at all
2
3
4
5
Perfectly
20
Q

A nurse at the local hospital undergoes vaccination against Hep B. 3 months after completion of the primary course:
Anti-HBs: 10 - 100 mIU/ml
(An antibody level of >100 mIU/ml indicates a good immune response with protective immunity)
What is the most appropriate course of action?
A) Repeat course (3 doses)
B) Repeat anti-HbS level in 3M
C) Give HbIG and one further dose of vaccine
D) Give one further dose of vaccine
E) Do a HIV test

A

Testing for anti-HBs is only recommended for those at risk of occupational exposure and patients with CKD. Anti-HBs levels should be checked 1-4 months after primary immunisation.

> 100 = adequate respone, booster in 5Y
10-100 = one additional vaccine dose should be given
<10 = Test for current or past infection. Give further vaccine course (i.e. 3 doses again) with testing following. If still fails to respond then HBIG would be required for protection if exposed to the virus.

How well did you know this?
1
Not at all
2
3
4
5
Perfectly
21
Q

What is the most appropriate initial management for a patient with suspected cord compression secondary to advanced, non-small cell lung cancer?
A) Broad-spec antibiotics
B) High dose PO dexamethasone
C) IV mannitol
D) Urgent MRI spine
E) Urgent referral for radiotherapy

A

Spinal cord compression is an oncological emergency and so dexamethasone should be given whilst arranging investigations, ideally a whole MRI spine within 24hours of presentation.

How well did you know this?
1
Not at all
2
3
4
5
Perfectly
22
Q

What organism is most likely to be responsible in causing IE in a patient with a recent dental extraction and poor dentition?
A) Eikenella corodens
B) MRSA
C) S. aureus
D) Streptococcus sanguinis
E) Streptococcus bovis

A

Patients with very poor dental hygiene may develop endocarditis secondary to Viridans streptococci e.g. Streptococcus sanguinis

The two most notable viridans streptococci are Streptococcus mitis and Streptococcus sanguinis - they are both commonly found in the mouth and in particular dental plaque so endocarditis caused by these organisms is linked with poor dental hygiene or following a dental procedure.

How well did you know this?
1
Not at all
2
3
4
5
Perfectly
23
Q

A 36F presents with skin lesions on her upper limbs. She has recently been on extensive travels throughout the world. OE she has an annular granulomatous rash with an asymmetrical distribution, 3 lesions on her left arm, and 1 lesion on her right arm. The lesions are hypopigmented and have well defined raised borders. They are associated with hair loss and decreased sensation. What is the most likely diagnosis?
A) Bullous pemphigoid
B) Cutaneous leishmaniasis
C) Lyme disease
D) Necrotising fasciitis
E) Tuberculoid leprosy

A

Tuberculoid leprosy is suggested by a limited number of skin lesions, hypaesthesia, and hair loss. Mycobacterium leprae are scanty in tuberculoid leprosy lesions. It is a relatively benign form of leprosy and one of the least contagious.

Cutaneous leishmaniasis is characterised by an initial small red papule that enlarges in size and develops a central ulceration that can exude pus or dry with a crusted scab.

How well did you know this?
1
Not at all
2
3
4
5
Perfectly
24
Q

A 67M with a history of HTN presents with a 24hr history of dyspnoea and palpitations. He also complains of mild chest discomfort. OE, you note an irregularly irregular pulse (115bpm), BP 95 / 70 mmHg and RR 20 breaths/min. Nil regular medication. An ECG shows absent P waves with QRS complexes irregularly irregular intervals. What is the most appropriate management?
A) Digoxin
B) B-blocker
C) IV adenosine
D) DCCV
E) Clopidogrel

A

BP of 95/70 mmHg in a patient with a history of HTN, who is currently not taking any BP medication is quite concerning. It suggests that he is hemodynamically unstable. New-onset AF within 48hrs should be treated with DC cardioversion if unstable or either DC cardioversion or pharmacological cardioversion.

If the AF is definitely of less than 48 hours onset patients should be heparinised. If the patient has been in AF for more than 48 hours then anticoagulation should be given for at least 3 weeks prior to cardioversion. Following electrical cardioversion patients should be anticoagulated for at least 4 weeks.

How well did you know this?
1
Not at all
2
3
4
5
Perfectly
25
Q

Which of the following cytotoxic chemotherapy agents are most likely to be responsible for low magnesium levels?
A) Docetaxel
B) Vincristine
C) Cisplatin
D) Cyclophosphamide
E) Cytarabine

A

Cisplatin is associated with hypomagnesaemia.

-Docetaxel (ovarian cancer) = neutropenia
-Vincristine (haematological malignancies and breast cancer) = peripheral neuropathy and paralytic ileus
-Cyclophosphamide (ovarian cancer) = haemorrhagic cystitis, myelosuppression and transitional cell carcinoma.
-Cytarabine (haematological malignancies) = myelosuppression and ataxia.

How well did you know this?
1
Not at all
2
3
4
5
Perfectly
26
Q

A 42M presents a two-month history of steatorrhoea, abdominal discomfort, and diarrhoea. 3 years ago, he required colonic resection following ischaemic colitis, leaving him with short bowel syndrome. He also reports paraesthesia and feeling unsteady on his feet. The endoscopic appearance of the small bowel is unremarkable. Biopsy samples show non-specific eosinophilia. Colonoscopy is also unremarkable. Abdominal examination is unremarkable, and his weight has remained stable. What investigation is most likely to be diagnostic?
A) CT abdomen
B) CT colonoscopy
C) Faecal calprotectin
D) Hydrogen breath test
E) Serum lipase

A

Small bowel overgrowth syndrome (SBOS) is where excessive microbes colonise the bowel. As in this case, anatomic disorders can predispose to SBOS due to intestinal stasis. In the majority of cases, it presents as bloating and steatorrhoea. In patients with vitamin B12 deficiency, neurological symptoms can also co-exist. The diagnosis can be established with a positive hydrogen breath test. The mucosal histology is variable, and may include modest villous blunting accompanied by increased lamina propria and epithelial inflammation. It is often normal.

How well did you know this?
1
Not at all
2
3
4
5
Perfectly
27
Q

What cancer is a male patient positive for the BRCA gene more likely to have?
A) Colon
B) Lung
C) Prostate
D) Retinoblastoma
E) Brain

A

Prostate cancer in males is strongly associated with the BRCA genes.

How well did you know this?
1
Not at all
2
3
4
5
Perfectly
28
Q

A 38F presents to ED with poorly-defined symptoms of headache and drowsiness. She has a PMHx of asthma and is known to be HIV-positive. An urgent CT head scan demonstrates a single brain lesion with homogenous enhancement. A subsequent thallium SPECT scan is conducted and has a positive result. Given the findings, what is the most likely diagnosis?
A) Encephalitis
B) Lymphoma
C) PML
D) Toxoplasmosis
E) Tb

A

The most likely finding here is primary CNS lymphoma, given the neurological symptoms in an HIV-positive patient and the single brain lesion with homogenous enhancement.

Treatment generally involves steroids, chemotherapy and possible surgical intervention.

Toxoplasmosis presents similarly but with multiple ring-enhancing lesions and is Thallium SPECT negative.

How well did you know this?
1
Not at all
2
3
4
5
Perfectly
29
Q

A 56M presents with falls. He feels he is getting progressively weaker and has difficulties walking up the stairs. He is hypertensive and has suffered with chronic smokers cough for years. O/E he has weakness of hip flexion, particularly knee flexion. He is unable to keep his fingers flexed against force, the right being weaker than the left. There are no sensory abnormalities and reflexes are preserved. Serum CK is normal.
A) Cervical cord compression
B) Inclusion body myositis (IBM)
C) Limb-girdle muscular dystrophy
D) MND
E) Polymyositis

A

IBM is an idiopathic inflammatory myopathy. The pattern of muscle weakness (quadriceps and long finger flexors) and asymmetry of signs is characteristic. IBM is the most common acquired myopathy in people >50 and occurs more often in men.

Limb-girdle muscular dystrophy usually presents with slowly progressive proximal arm/leg weakness. It is rare.

How well did you know this?
1
Not at all
2
3
4
5
Perfectly
30
Q

A patient presents with bradycardia and profound hypotension. It is thought to be due to b-blocker overdose. Which of the following agents is most suitable in initial treatment?
A) Adrenaline
B) Glucagon
C) Insulin
D) Neostigmine
E) Pyridostigmine

A

The initial treatment of b-blocker overdose is atropine but the preferred treatment of patients with profound bradycardia and hypotension is IV bolus of glucagon followed by IV glucagon infusion.

Adrenaline and isoprenaline can be given if glucagon is not available.

How well did you know this?
1
Not at all
2
3
4
5
Perfectly
31
Q

Following a stroke/TIA, what driving restrictions apply?
A) Must not drive a heavy goods vehicle for at least 2 years
B) Must not drive for 1M
C) Must not drive for 3M
D) Must not drive until neurological signs are resolved
E) No driving necessary

A

Strokes and TIA have driving restrictions for 1M from the date of attack. If the significant neurological deficits that interfere with the ability to drive persists the the restriction is likely to be prolonged.

How well did you know this?
1
Not at all
2
3
4
5
Perfectly
32
Q

Which of the following abnormalities would be expected to be seen in a patient with refeeding syndrome?
A) Raised LH and FSH
B) Elevated circulating cortisol
C) Low resting growth hormone level
D) Increased levels of GnRH
E) Normal oestrogen levels

A

Circulating cortisol have been shown to be elevated in anorexia.

How well did you know this?
1
Not at all
2
3
4
5
Perfectly
33
Q

An unwell patient is found to have P. falciparum species seen, 12% parasitaemia on malarial films. What treatment should be commenced?
A) Chloroquine
B) Artesunate and exchange transfusion
C) Artesunate
D) Doxycycline
E) Quinine and doxycycline

A

Patients with severe malaria should be treated with IV artesunate, and in cases where parasitaemia >10% is seen, consideration should be given to the performance of exchange transfusions.

How well did you know this?
1
Not at all
2
3
4
5
Perfectly
34
Q

54M with T2DM is found to have new vessel formation at the optic disc. Visual acuity in both eyes is not affected (6/9). Blood pressure is155/84 mmHg. HbA1c: 68 mmol/mol (8.4%) What is the most important intervention in this patient?
A) Follow up ophthalmoscopy in 3M
B) Add aspirin
C) BP control
D) Tight glycemic control
E) Laser therapy

A

This patient has proliferative diabetic retinopathy and urgent referral to an ophthalmologist for panretinal photocoagulation is indicated. Intravitreal VEGF inhibitors can also be used alongside laser therapy.

How well did you know this?
1
Not at all
2
3
4
5
Perfectly
35
Q

A 65M has been started on amiodarone. He has been told he must first take higher doses and then continue on a lower maintenance dose long-term. What is the reasoning behind this initial dose regime?
A) Extensive hepatic P450 breakdown of amiodarone
B) Fast metabolism of amiodarone due to extensive protein binding
C) Slow absorption of amiodarone via the enteral route
D) Slow metabolism of amiodarone due to extensive protein binding
E) Slow renal excretion of amiodarone

A

Slow metabolism of amiodarone due to extensive lipid binding is the correct answer. Lipid-bound drugs generally have longer half-lives as they are extensively distributed in tissues, primarily adipocytes. A longer half-life means that loading doses are required. The time taken to achieve stable systemic drug concentration correlates with the half-life of that drug. A long half-life means a drug is cleared more slowly. If the patient were to start taking a drug at the maintenance dose it would take much longer to reach therapeutic concentrations.

How well did you know this?
1
Not at all
2
3
4
5
Perfectly
36
Q

A 20F Nigerian presents with chest pain. She reports a long history of fatigue and joint pains. Examination reveals a pericardial rub and bilateral pitting oedema. Observations show only that she has a low grade pyrexia 37.7ºC and BP 170/100 mmHg. Urinalysis shows haematuria and nephrotic-range proteinuria. ECG shows saddle-shaped ST-elevation in all leads. Bloods: Urea 8.2 mmol/l, Creatinine 212 µmol/l, eGFR 33ml/m/m². Which of the following histopathological findings is most likely on renal biopsy?
A) Focal proliferative glomerulonephritis
B) Membranous glomerulonephritis
C) Diffuse proliferative glomerulonephritis
D) FSGS
E) Mesangial proliferative glomerulonephritis

A

The presence of hypertension, kidney injury, proteinuria and haematuria point towards a nephritic picture of kidney insult.

The multisystem presentation of fever, arthralgia, pericarditis and nephritis associated with the epidemiological clues (a young black female) suggests SLE.

The most common histological pattern seen in lupus nephritis is diffuse proliferative glomerulonephritis.

Class IV (diffuse proliferative glomerulonephritis) is the most common and severe of the glomerulonephritides. Renal biopsy characteristically shows:
-glomeruli shows endothelial and mesangial proliferation, ‘wire-loop’ appearance
-if severe, the capillary wall may be thickened secondary to immune complex deposition
-electron microscopy shows subendothelial immune complex deposits
-granular appearance on immunofluorescence

How well did you know this?
1
Not at all
2
3
4
5
Perfectly
37
Q

Which of the following medications are most associated with constipation?
A) Verapamil
B) Sitagliptin
C) Metformin
D) Ramipril
E) Atorvastatin

A

Verapamil, like other calcium channel blockers, is a common pharmacological cause of constipation.

How well did you know this?
1
Not at all
2
3
4
5
Perfectly
37
Q

Which of the following advice should be given to a patient on carbamazepine who wants to start breastfeeding?
A) Breast-feeding should be avoided whilst taking carbamazepine
B) Increase the dose of carbamazepine
C) No changes required
D) Switch carbamazepine to lamotrigine
E) Switch carbamazepine to levetiracetem

A

Breast feeding is acceptable with nearly all anti-epileptic drugs.

How well did you know this?
1
Not at all
2
3
4
5
Perfectly
38
Q

Which foramen does the maxillary nerve go through?
A) Jugular foramen
B) Foramen ovale
C) Superior orbital fissure
D) Optic canal
E) Foramen rotundum

A

Foramen rotundum

How well did you know this?
1
Not at all
2
3
4
5
Perfectly
39
Q

A 34M is investigated following an unexplained collapse whilst at work. A resting ECG shows convex ST elevation in V1-V3 with a partial right bundle branch block pattern. What is the most likely diagnosis?
A) Polymorphic VT
B) HOCM
C) Arrythmogenic right ventricular cardiomyopathy
D) Brugada syndrome
E) Normal variant

A

Brugada syndrome is a form of inherited CV disease which may present with sudden cardiac death. It is an autosomal dominant disease and is more common in Asians.

ECG changes
-convex ST segment elevation > 2mm in > 1 of V1-V3 followed by a negative T wave
-partial right bundle branch block

The ECG changes may be more apparent following the administration of flecainide or ajmaline (investigation of choice)

How well did you know this?
1
Not at all
2
3
4
5
Perfectly
40
Q

A research facility is trying to develop a new test to screen for breast cancer compared to current standards which include mammography. From preliminary data, the new screening test appears to be better at picking up early cancers. When comparing both these tests, however, there appears to be no obvious difference between survival rates. What is this an example of?
A) Lead-look bias
B) Lead-time bias
C) Publication bias
D) Recall bias
E) Work up bias

A

Lead-time bias - occurs when two tests for a disease are compared, the new test diagnoses the disease earlier, but there is no effect on the outcome of the disease

How well did you know this?
1
Not at all
2
3
4
5
Perfectly
41
Q

A fit and well 24M presents with palpitations in the early hours of the morning after consuming a large amount of alcohol. He denied any chest pain or breathlessness. His pulse was 120 bpm and irregularly irregular. His BP was 124/70 mmHg. ECG showed AF with a ventricular rate of 118. What is the most appropriate initial management?
A) Oral bisoprolol
B) IV metoprolol
C) Oral flecainide
D) ECV
E) IVF

A

This patient has atrial fibrillation with a fast ventricular rate but is otherwise stable and shows no signs of decompensation. This is likely to have been precipitated by his heavy alcohol consumption and it is reasonable to rehydrate him with IVF and see whether this reverts him to sinus rhythm before attempting pharmacological or electrical cardioversion.

How well did you know this?
1
Not at all
2
3
4
5
Perfectly
42
Q

An 80M presents with recurrent episodes of hallucinations. He describes seeing faces smaller than normal or other objects out of proportion. He finds these episodes distressing although he says he knows they’re not real. PMHx of macular degeneration and an episode of depression 20 years ago following the death of his wife. Neurological examination is unremarkable. What is the most likely diagnosis?
A) Schizophrenia
B) Charles-Bonnet syndrome
C) Psychotic depression
D) Cerebral tumour
E) Alzheimers disease

A

Charles-Bonnet syndrome is characterised by persistent or recurrent complex hallucinations (usually visual or auditory), occurring in clear consciousness. This is generally against a background of visual impairment. Insight is usually preserved. This must occur in the absence of any other significant neuropsychiatric disturbance.

How well did you know this?
1
Not at all
2
3
4
5
Perfectly
43
Q

A 41M is diagnosed with idiopathic pulmonary arterial hypertension. He undergoes vasodilator testing with intravenous epoprostenol to which he has a positive response. What is the most appropriate first-line treatment for his condition?
A) Sildenafil
B) Trepostinil
C) Bosentan
D) ISMN
E) Nifedipine

A

Patients who test positive to vasodilator testing are typically treated with calcium channel blockers such as nifedipine, diltiazem and increasingly amlodipine.

If there is a negative response to acute vasodilator testing (the majority) –>
-prostacyclin analogues: treprostinil, iloprost
-endothelin receptor antagonists: bosentan, ambrisentan
-phosphodiesterase inhibitors: sildenafil

How well did you know this?
1
Not at all
2
3
4
5
Perfectly
44
Q

What would you expect the normal anatomical dead space to be in a healthy adult male?
A) 50ml
B) 150ml
C) 250ml
D) 350ml
E) 450ml

A

150ml

How well did you know this?
1
Not at all
2
3
4
5
Perfectly
45
Q

A cocaine user presents to ED with multiple superficial lacerations. She says she can feel something crawling under her skin and was trying to see what was there. What is this symptom known as?
A) Auditory hallucination
B) Visual hallucination
C) Haptic hallucination
D) Primary delusion of infestation
E) Illusion

A

Haptic hallucinations are experiences of sensations of being touched/strangled/feeling of insects beneath skin. These can often occur in cocaine users. Haptic hallucinations also occur in alcohol withdrawal and schizophrenia.

How well did you know this?
1
Not at all
2
3
4
5
Perfectly
46
Q

Which of the following prednisolone doses matches most closely with the physiological production of glucocorticoid in a 75kg male?
A) 1mg
B) 2mg
C) 7.5mg
D) 15mg
E) 20mg

A

7.5mg

How well did you know this?
1
Not at all
2
3
4
5
Perfectly
47
Q

Which of the following factors increases the risk of developing hydralazine-induced lupus?
A) White ethnicity
B) dsDNA +ve
C) Family Hx of SLE
D) HLA-B6 genotype
E) Rapid acetylator status

A

SLE is more common in Afro-caribbeans whilst drug-induced SLE is more common in those of white ethnicity, men and women affected equally. the reaction is dose and concentration-dependent.

Slow acetylator status increases the risk of developing drug-induced SLE due the slower metabolism leading to elevated concentrations in the blood. Drug-induced lupus is more common in those with HLA-DR4 genotype.

How well did you know this?
1
Not at all
2
3
4
5
Perfectly
48
Q

For a patient with discrete small cell lung cancer, which of the following systemic therapies is most appropriate to use alongside radiotherapy?
A) Cisplatin and etoposide
B) Carboplatin and etoposide
C) Carboplatin and paclitaxel
D) Topotecan and ifosfamide
E) Vinorelbine and cisplatin

A

Cisplatin and etoposide is standard therapy for small cell lung cancer, but carboplatin is frequently used in patients who can’t tolerate it or have a contra-indication to it.

How well did you know this?
1
Not at all
2
3
4
5
Perfectly
49
Q

Which of the following is accurate regarding the pharmacological interaction between lithium and anti-hypertensive agents?
A) Acetazolamide decreases lithium concentrations
B) ACEi leads to decreased lithium concentration
C) Bendroflumethiazide decreases lithium concentration
D) CCB lessen the risk of lithium neurotoxicity
E) Methyldopa leads to decreased lithium neurotoxicity

A

Osmotic diuretics and carbonic anhydrase inhibitors such as acetazolamide lead to increased renal excretion of sodium and lithium.

How well did you know this?
1
Not at all
2
3
4
5
Perfectly
50
Q

Which of the following tissues are most commonly affected in SLE?
A) Joints
B) Skin
C) Kidneys
D) Lungs
E) Heart and blood vessels

A

Joints (80%). 75% of patients have skin manifestations.

How well did you know this?
1
Not at all
2
3
4
5
Perfectly
51
Q

Which of the following blood results is most consistent with haemachromatosis?
A) ALP 178 (45-105)
B) Ferritin 324 (15-300)
C) Serum iron 25 (12-30)
D) TIBC 43 (45-75)
E) Transferrin saturation 78% (20-50)

A

Haemachromatosis is caused by a mutation in the gene HFE which results in iron accumulation in different organs. Findings suggestive of iron accumulation:
-Ferritin >500
-Serum iron >30
-Transferrin saturation >60%
-TIBC <20

How well did you know this?
1
Not at all
2
3
4
5
Perfectly
52
Q

What is the treatment for Ramsay Hunt syndrome?
A) Corticosteroids only
B) IV aciclovir
C) PO aciclovir and corticosteroids
D) PO aciclovir
E) Reassurance

A

Full recovery is more likely if antiviral treatment is started within 72 hours of the onset of symptoms. First-line treatment includes aciclovir 800 mg orally five times daily for 7 days and prednisone 60 mg orally daily for 5 days.

Steroids are helpful due to their strong anti-inflammatory action. This is thought to reduce oedema in the affected nerves and result in shorter disease duration.

How well did you know this?
1
Not at all
2
3
4
5
Perfectly
53
Q

What feature on examination most strongly suggests that the mitral valve leaflets are still mobile?
A) Loud S1
B) Loud diastolic murmur
C) Opening snap
D) Quiet S1
E) Quiet diastolic murmur

A

In mitral stenosis, an opening snap indicates the leaflets still have some mobility.

An opening snap is pathognomonic of mitral valve stenosis. It is a high-pitched early diastolic sound (just after S2) due to the sudden contraction of the valve leaflets after their initial opening. A clear opening snap suggests that the valve is pliable and would therefore be suitable for balloon mitral valvuloplasty.

Features of severe MS
length of murmur increases
opening snap becomes closer to S2.

How well did you know this?
1
Not at all
2
3
4
5
Perfectly
54
Q

A 54M with a history of depression presents for review. He was started on fluoxetine 8W ago and is now requesting to stop his medication as he feels so well. What should be recommended regarding his treatment?
A) It should be stopped straight away
B) It should be continued for another 6W
C) It should be continued for another 3M
D) It should be continued for another 6M
E) It should be continued for another 12M

A

Antidepressants should be continued for at least 6 months after remission of symptoms to decrease risk of relapse.

When stopping a SSRI the dose should be gradually reduced over a 4 week period (this is not necessary with fluoxetine).

How well did you know this?
1
Not at all
2
3
4
5
Perfectly
55
Q

What is the causative organism of granuloma inguinale?
A) Klebsiella granulomatis
B) Chlamydia
C) HSV
D) Treponema pallidum
E) Haemophilus ducreyi

A

Granuloma inguinale - Klebsiella granulomatis

Chancroid = caused by Haemophilus ducreyi. It causes painful genital ulcers associated with unilateral, painful inguinal lymph node enlargement. The ulcers typically have a sharply defined, ragged, undermined border.

Lymphogranuloma venereum (LGV) = Chlamydia trachomatis. Typically infection comprises of three stages
-stage 1: small painless pustule which later forms an ulcer
-stage 2: painful inguinal lymphadenopathy
-stage 3: proctocolitis

How well did you know this?
1
Not at all
2
3
4
5
Perfectly
56
Q

What is the mechanism of action of procyclidine?
A) Anti-muscarinic
B) Dopamine receptor agonist
C) Decarboxylase inhibitor
D) Dopamine receptor antagonist
E) MAO-B inhibitor

A

Procyclidine - antimuscarinic

How well did you know this?
1
Not at all
2
3
4
5
Perfectly
57
Q

In the Gell and Coombs classification of hypersensitivity reactions Grave’s disease is an example of a:
A) Type I reaction
B) Type II reaction
C) Type III reaction
D) Type IV reaction
E) Type V reaction

A

Type V = Antibodies that recognise and bind to the cell surface receptors.

This either stimulating them or blocking ligand binding
-Graves’ disease
-Myasthenia gravis

How well did you know this?
1
Not at all
2
3
4
5
Perfectly
58
Q

The first-line treatment in remnant hyperlipidaemia (dysbetalipoproteinaemia) is:
A) Ursodeoxycholic acid
B) Vitamin A
C) Statins
D) Fish oil
E) Fibrates

A

Fibrates are first line treatment

How well did you know this?
1
Not at all
2
3
4
5
Perfectly
59
Q

A 63M is noted to have a pleural effusion on CXR. Which one of the following would typically cause a transudate?
A) Pancreatitis
B) Pneumonia
C) Yellow-nail syndrome
D) Hypothyroidism
E) Dresslers syndrome

A

Transudate (< 30g/L protein)
-heart failure
-hypoalbuminaemia
-liver disease
-nephrotic syndrome
-malabsorption
-hypothyroidism
-Meigs’ syndrome

Exudate (> 30g/L protein)
-infection
-pneumonia
-tuberculosis
-subphrenic abscess
-connective tissue disease
-rheumatoid arthritis
-systemic lupus erythmatosus
-neoplasia
-lung cancer
-mesothelioma
-metastases
-pancreatitis
-pulmonary embolism
-Dressler’s syndrome
-yellow nail syndrome

How well did you know this?
1
Not at all
2
3
4
5
Perfectly
60
Q

What does the power of the study equate to?
A) 1/p value
B) SD/square root of sample size
C) 1-probability of making Type II error
D) 1-probability of making Type I error
E) 1/probability of making Type I error

A

Power = 1 - the probability of a type II error

How well did you know this?
1
Not at all
2
3
4
5
Perfectly
61
Q

What is the main advantage of using capecitabine instead of fluorouracil?
A) Current data shows increased survival
B) Less cardiotoxic
C) PO administration
D) Less nausea
E) Not really excreted so can be used in patients with CKD

A

Capecitabine is an orally administered prodrug which is enzymatically converted to 5-fluorouracil in the tumour.

How well did you know this?
1
Not at all
2
3
4
5
Perfectly
62
Q

How does C. difficile spread?
A) Aerosol via ingestion of spores
B) Aerosol via ingestion of toxins
C) Faeco-oral via ingestion of spores
D) Faeco-oral via ingestion of toxins
E) Needle-stick injury

A

Clostridioides difficile: spreads via the faecal-oral route by ingestion of spores. Toxin A and B are responsible for the clinical manifestations of Clostridioides difficile infection but not its transmission.

How well did you know this?
1
Not at all
2
3
4
5
Perfectly
63
Q

Which part of the ECG does the S4 heart sound coincide with?
A) P wave
B) QRS complex
C) ST segment
D) T wave
E) U wave

A

The S4 heart sound is caused by atrial contraction against a stiff ventricle, occurring just before the S1 sound. It may be heard in aortic stenosis, hypertrophic cardiomyopathy or hypertension. As such, it coincides with the P wave, which represents atrial depolarisation.

How well did you know this?
1
Not at all
2
3
4
5
Perfectly
64
Q

What is the mechanism of action of timolol for use in primary open-angle glaucoma?
A) Increased uveoscleral outflow
B) Pupil constriction
C) Pupil dilation
D) Reduces aqueous production
E) Reduces aqueous production and increases uveoscleral outflow

A

Timolol is a beta-blocker which decreases intraocular pressure by reducing production of aqueous humour.

Prostaglandin analogues (latanoprost) are first-line agents in primary open-angle glaucoma and work by increasing uveoscleral outflow, but they do not reduce aqueous production. Miotics (pilocarpine) work on the muscarinic receptor causing pupil constriction and therefore increase uveoscleral outflow.

How well did you know this?
1
Not at all
2
3
4
5
Perfectly
65
Q

Which one of the following best describes the characteristics of a negatively skewed distribution?
A) Median < mean < mode
B) Mean < median < mode
C) Mode < mean < median
D) Median < mode < mean
E) Mean < mode < median

A

Skewed distributions
alphabetical order: mean - median - mode
‘>’ for positive, ‘<’ for negative

How well did you know this?
1
Not at all
2
3
4
5
Perfectly
66
Q

Which of the following analyses is most useful when comparing different treatment therapies?
A) Completer analysis
B) Intention to treat analysis
C) Per-protocol analysis
D) Screened and randomised analysis
E) Sensitivity analysis

A

Intention to treat analysis involves looking at all the people taking part in a trial based on the group they were allocated to regardless of whether or not they actually took the proposed treatment. Intention to treat analysis are often used to assess clinical effectiveness as they mirror actual clinical practice and is hence a robust way of monitoring the effectiveness of therapy.

How well did you know this?
1
Not at all
2
3
4
5
Perfectly
67
Q

Which of the following antibiotics is the best choice to treat typhoid fever in a patient who is 25W pregnant?
A) Ceftriaxone
B) Chloramphenicol
C) Ciprofloxacin
D) Co-trimoxazole
E) Ofloxacin

A

All these antibiotics can be used to treat typhoid fever. Ceftriaxone has the best safety profile in pregnancy.

Quinolones are not recommended in pregnancy.

How well did you know this?
1
Not at all
2
3
4
5
Perfectly
68
Q

A 75M presents with pedal oedema and ++ protein in his urine. He has a PMHx of SqCC of the lung which was resected. What is the most likely diagnosis?
A) IgA nephropathy
B) Membranous nephropathy
C) Minimal change disease
D) FSGS
E) Membranoproliferative glomerulonephritis

A

Membranous nephropathy is often idiopathic but can be triggered by malignancy of the bowel and bronchus, SLE, penicillamine therapy and hep B infection. There is a strong association with HLA-DR3. M>F. Thickening of the capillary BM due to immune complex deposition is the main feature of membranous nephropathy. 1/3 of patients develop CKD within 10-20Y.

How well did you know this?
1
Not at all
2
3
4
5
Perfectly
69
Q

A man presents with short stature, short 5th metacarpals, subcutaneous calcification, intellectual impairment and hypocalcaemia. Given the likely diagnosis, where is the most likely defect?
A) A G-protein linked to PTHr
B) A serine kinase receptor
C) A tyrosine kinase receptor
D) A tyrosine phosphatase receptor
E) An ion channel linked to the PTHr

A

This patient has pseudohypoparathyroidism, related to a loss-of-function mutation affecting the G-protein linked receptor for PTH.

How well did you know this?
1
Not at all
2
3
4
5
Perfectly
70
Q

Which of the following investigative findings on urine examination would be most consistent with acute tubular necrosis?
A) Visible haematuria
B) Myoglobin casts
C) Red cell casts
D) Urine osmolality 300mosmol/kg
E) Urine sodium 20mmol/l

A

Urine osmolality decreases in ATN, reflecting reduced tubal function and filtration rate. This is in contrast to pre-kidney disease with urine osmolality is high due to poor urine output and high creatinine.

How well did you know this?
1
Not at all
2
3
4
5
Perfectly
71
Q

Which of the following potential consequences are associated with the presence of maternal thyroid autoantibodies?
A) Higher risk of spontaneous abortion
B) Increased risk of neonatal hypothyroidism
C) Increased risk of pregnancy-induced hypertension
D) Increased risk of macrosomia
E) Increased risk of transient neonatal thyrotoxicosis

A

The presence of maternal thyroid autoantibodies is strongly associated with miscarriage and pre-term delivery.

How well did you know this?
1
Not at all
2
3
4
5
Perfectly
72
Q

Which of the following investigations would confirm a PDA?
A) CXR
B) 12 lead ECG
C) 2D Echo
D) Carotid doppler study
E) TOE with bubble study

A

-Incomplete RBBB may be seen on ECG
-Echo may demonstrate the defect and show right ventricular dilatation and hypertrophy and pulmonary artery dilatation but PFOs may be missed on 2D echo
-The precise size and location of the defect can be shown on TOE.

A 2D echo cannot exclude a PFO unless a bubble study and Valsalva are performed as a PFO is a potential space that is only open during Valsalva conditions.

How well did you know this?
1
Not at all
2
3
4
5
Perfectly
73
Q

Which hormone is chiefly responsible for epiphyseal fusion and cessation of growth?
A) GH
B) Testosterone
C) Oestrogen
D) Thyroxine
E) Somatostatin

A

The growth spurt at puberty is brought about by the secretion of androgens in males and oestrogen in females; however it is oestrogens that ultimately terminate growth by causing the epiphyses in the long bones to fuse.

How well did you know this?
1
Not at all
2
3
4
5
Perfectly
74
Q

Which of the following diuretics is most likely to be responsible for a K 3.4, acidosis and serum bicarb 15mmol?
A) Acetazolamide
B) Amiloride
C) Bumetanide
D) Furosemide
E) Metolazone

A

Acetazolamide inhibits proximal tubular bicarbonate reabsorption in a similar fashion to Type 2 RTA.

How well did you know this?
1
Not at all
2
3
4
5
Perfectly
75
Q

TWhat is the best treatment option for a patient with a large right-sided pneumothorax, midline shift and hypotension?
A) Chest drain insertion
B) Chest drain insertion under radiographic control
C) Needle aspiration in left mid-axillary line
D) Repeat CXR after a few hours
E) Needle aspiration of the right pleural space

A

The most appropriate management would be needle decompression in either the 2nd ICS mid clavicular line or 5th ICS mid axillary line followed later by chest drain insertion.

How well did you know this?
1
Not at all
2
3
4
5
Perfectly
76
Q

Which HLA gene is most strongly associated with coeliac disease?
A) HLA-DQ2
B) HLA-B27
C) HLA-DQ4
D) HLA-DR4
E) HLA-C13

A

Coeliac disease has a strong association with HLA-DQ2 (present in 95% of patients).

How well did you know this?
1
Not at all
2
3
4
5
Perfectly
77
Q

Which one of the following is not a recognised feature of methaemoglobinaemia?
A) Chocolate cyanosis
B) Dyspnoea
C) Anxiety
D) Acidosis
E) Reduced pO2 but normal sats on pulse oximetry

A

Normal pO2 but decreased oxygen saturation is characteristic of methaemoglobinaemia.

How well did you know this?
1
Not at all
2
3
4
5
Perfectly
77
Q

Which one of the following is not a recognised feature of methaemoglobinaemia?
A) Chocolate cyanosis
B) Dyspnoea
C) Anxiety
D) Acidosis
E) Reduced pO2 but normal sats on pulse oximetry

A

Normal pO2 but decreased oxygen saturation is characteristic of methaemoglobinaemia.

How well did you know this?
1
Not at all
2
3
4
5
Perfectly
78
Q

Which one of the following features would be most consistent with a diagnosis of Bell’s palsy?
A) Vesicular rash around the ear
B) Hyperacusis
C) Sensory loss over distribution of the facial nerve
D) Pins and needles in R arm
E) Rhinorrhoea

A

A vesicular rash around the ear would suggest a diagnosis of Ramsey Hunt syndrome. Hyperacusis is seen in around a third of patients.

Patients may also notice
-post-auricular pain
-altered taste
-dry eyes
-hyperacusis

How well did you know this?
1
Not at all
2
3
4
5
Perfectly
79
Q

What is the most appropriate treatment of a severely symptomatic patient with a Na of 109 mmol/l?
A) Bolus of hypertonic saline
B) Bolus of normal saline
C) Infusion of loop diuretic
D) Infusion of hypertonic saline over 8h
E) Infusion of normal saline over 8h

A

Severe hyponatraemia is treated with boluses of hypertonic saline until symptoms have resolved.

Infusions of hypertonic saline over 8 hours may be useful in moderate hyponatraemia however due to the severity of the hyponatraemia, this is not a sufficient rate in this case.

How well did you know this?
1
Not at all
2
3
4
5
Perfectly
80
Q

Why is the COCP contra-indicated in a patient with migraines with aura?
A) Theoretical risk of ischaemic stroke
B) Significantly increased risk of ischaemic stroke
C) Increased frequency of migraines
D) Migraine is an independent RF of VTE
E) Increased severity of migraines

A

Significantly increased risk of ischaemic stroke

Migraine during pregnancy:
-paracetamol 1g = first-line
-NSAIDs can be used second-line in the first and second trimester
-avoid aspirin and opioids such as codeine during pregnancy

How well did you know this?
1
Not at all
2
3
4
5
Perfectly
81
Q

A 24F presents due to a rash on her neck and forehead. She returned from a holiday in Cyprus 1 week ago and had her hair dyed 2 days ago. On examination there is a weepy, vesicular rash around her hairline although the scalp itself is not badly affected. What is the most likely diagnosis?
A) Cutaneous leishmaniasis
B) Irritant contact dermatitis
C) Allergic contact dermatitis
D) Syphilis
E) Photocontact dermatitis

A

Allergic contact dermatitis

-2 main types of contact dermatitis:
-irritant contact dermatitis: common - non-allergic reaction due to weak acids or alkalis (e.g. detergents). Often seen on the hands. Erythema is typical, crusting and vesicles are rare
-allergic contact dermatitis: type IV hypersensitivity reaction. Uncommon - often seen on the head following hair dyes. Presents as an acute weeping eczema which predominately affects the margins of the hairline rather than the hairy scalp itself. Topical treatment with a potent steroid is indicated

How well did you know this?
1
Not at all
2
3
4
5
Perfectly
82
Q

A 24M has a cardiac arrest and the defibrillator indicates PEA. The patient was given 1mg of IV medication and chest compressions were continued. What type of receptor does this drug act on?
A) G-protein coupled receptor
B) Enzyme-linked receptor
C) Internal receptors
D) Ion channel-linked receptors
E) Voltage-sensitive ion channel

A

The drug which has been administered is adrenaline. Adrenaline binds to G protein-coupled receptors. G protein-coupled receptors, (seven-transmembrane domain receptors) are in the cell membrane and bind to extracellular molecules and transmit signals intracellularly. When a ligand binds to the G-protein coupled receptor, it causes a conformational change in the receptor. This then causes an associated G protein to become activated by exchanging GDP for GTP. The two principal signalling pathways involved in G protein-coupled receptors intracellularly are cyclic AMP (cAMP) pathway and the phosphatidylinositol pathway.

How well did you know this?
1
Not at all
2
3
4
5
Perfectly
83
Q

What treatment should be commenced in a patient who has been acutely treated for Plasmodium vivax infection?
A) ACT
B) Artesunate
C) Atovaquone and proguanil
D) Chloroquine
E) Primaquine

A

Non-falciparum malaria Plasmodium vivax and Plasmodium ovale can produce liver hypnozoites as part of their infection cycle. These hypnozoites can evade initial treatment by remaining dormant in the liver, before continuing the infection cycle after treatment has been stopped resulting in a relapse infection. Primaquine destroys these liver hypnozoites and therefore is used post-acute/initial therapy.

Chloroquine is used in first-line treatment for Plasmodium vivax and Plasmodium ovale, but must they again must be followed by primaquine.

How well did you know this?
1
Not at all
2
3
4
5
Perfectly
84
Q

What diagnostic test is recommended for a patient presenting first-line with HIV?
A) CD4
B) HIV-NAAT
C) HIV-1/2 Ab/Ag Immunoassay
D) P24
E) Western blot

A

HIV-1/2 Ab/Ag Immunoassay (fourth generation) should be used to screen individuals for HIV as it will detect HIV-1 and HIV-2 antibodies (Ab) and antigens (Ag) with high sensitivity within a short window period (approximately 10 days after infection).

If the initial screening results are not reactive or indeterminate and acute HIV infection is suspected then HIV-1 NAAT or p24 testing should be considered.

How well did you know this?
1
Not at all
2
3
4
5
Perfectly
85
Q

Which side-effect is most common following tetracycline treatment?
A) Headache
B) Red rash on face and neck
C) Dizziness
D) Dry lips and tongue
E) Ringing in ears

A

Tetracyclines are often prescribed for acne and can cause a photosensitive skin rash. This appears as a red rash on areas of skin exposed to the sun. Other skin reactions to tetracyclines include exfoliative dermatitis and Stevens-Johnson syndrome.

How well did you know this?
1
Not at all
2
3
4
5
Perfectly
86
Q

A 72M is admitted with chest pain. He is noted to be pale, have a HR of 40/min and a BP of 90/60 mmHg. Which one of the coronary arteries is most likely to be affected?
A) Posterior descending
B) LAD
C) Right coronary
D) Anterior interventricular
E) Left circumflex

A

This patient has developed complete heart block secondary to a right coronary artery (RCA) infarction. The atrioventricular node is supplied by the posterior interventricular artery, which in the majority of patients is a branch of the right coronary artery. In the remainder of patients the posterior interventricular artery is supplied by the left circumflex artery.

How well did you know this?
1
Not at all
2
3
4
5
Perfectly
87
Q

Which of the following medications are most likely to contribute towards hypercalcaemia?
A) Amlodipine
B) Enalapril
C) Bendroflumethiazide
D) Clopidogrel
E) Atorvastatin

A

Thiazides are recognised to cause hypercalcaemia but may also cause hypokalaemia, hypomagnesaemia, hyponatraemia and hypochloraemic alkalosis.

How well did you know this?
1
Not at all
2
3
4
5
Perfectly
88
Q

What is the MOA of adenosine?
A) A-blocker
B) B-blocker
C) G-protein coupled receptor agonist
D) Potassium channel blocker
E) Sodium channel blocker

A

G-protein coupled receptor agonist

How well did you know this?
1
Not at all
2
3
4
5
Perfectly
89
Q

Which of the following is most likely to be associated with episodes of facial angioedema?
A) Amlodipine
B) Aspirin
C) Bendroflumethiazide
D) Ramipril
E) Moxonidine

A

ACEi are associated with angioneurotic oedema and is a common side effect.

How well did you know this?
1
Not at all
2
3
4
5
Perfectly
90
Q

Which of the following antibodies are most likely to be positive in polymyositis?
A) Anti-smooth muscle
B) Anti-nuclear
C) Anti-dsDNA
D) Anti-SRP
E) pANCA

A

ANAs are positive in 1/3 of patients with polymyositis. Anti-Jo antibodies are also commonly seen.

How well did you know this?
1
Not at all
2
3
4
5
Perfectly
91
Q

Which of the following vaccinations should a patient with COPD get annually?
A) Clostridium tetani
B) H. influenzae type B
C) Meningococcus C
D) Influenza virus
E) Streptococcus pneumoniae

A

Influenza vaccine

How well did you know this?
1
Not at all
2
3
4
5
Perfectly
92
Q

A 25F presents complaining of various physical symptoms over the past 2 years. Examination is entirely normal. How would you best describe her condition?
A) Personality disorder
B) Somatisation disorder
C) Hypochondriacal disorder
D) Dissociative disorder
E) Depersonalisation

A

Somatisation disorder is characterised by multiple, changing physical symptoms. It requires the presence of:
-2 or more years of multiple, variable complaints
-Persistent refusal to accept the advice of several doctors
-Impairment of functioning due to symptoms and resultant behaviour

Hypochondriacal disorder is the preoccupation of having a serious disease.

How well did you know this?
1
Not at all
2
3
4
5
Perfectly
93
Q

A 67F with iron deficient anaemia is found to have an AS murmur radiating to both carotids. Upper GI endoscopy and colonoscopy are normal. What is the most appropriate next line investigation?
A) Barium enema
B) Bone marrow examination
C) Repeat colonoscopy
D) Repeat upper GI endoscopy
E) Selective mesenteric angiography

A

The most likely diagnosis is angiodysplasia - the most common vascular lesion of the GI tract. The condition may be asymptomatic or cause GI bleeding. The most commonly affected site is the large bowel. Angiodysplasia is associated with aortic stenosis.

Identification of colonic angiodysplasia is only 80% by colonoscopy, hence it is advisable to repeat colonoscopy.

How well did you know this?
1
Not at all
2
3
4
5
Perfectly
94
Q

Which of the following is the strongest pointer to T1DM?
A) Anti-GAD Ab
B) Age
C) BMI
D) Nocturia
E) Ketonuria

A

Ketonuria is indicative of insulinopenia and hence a strong indicator of T1DM.

Single autoantibody positivity (anti-GAD Ab) is not necessarily indicative of a diagnosis. 2 or 3 antibody positivity is much more indicative.

How well did you know this?
1
Not at all
2
3
4
5
Perfectly
94
Q

How is the NNT calculated over the duration of a study?
A) Number of months divided by the absolute risk reduction
B) Ratio of absolute to relative risk reduction
C) Ratio of relative to absolute risk reduction
D) Reciprocal of absolute risk reduction
E) Reciprocal of relative risk reduction

A

The NNT is calculated by 1/ARR

ARR = standard treatment events - new treatment events

RRR = ARR/evetns in control group

How well did you know this?
1
Not at all
2
3
4
5
Perfectly
95
Q

Which gene is likely implicated in a patient with a family history of breast cancer, ovarian cancer and melanoma?
A) BRCA1
B) BRCA2
C) APC
D) NF1
E) Rb

A

A family history of breast and ovarian cancer suggests a mutation in the BRCA genes. The family history of melanoma favours a mutation in BRCA2.

How well did you know this?
1
Not at all
2
3
4
5
Perfectly
96
Q

Which of the following is likely to precipitate digoxin toxicity?
A) Prolongation of PR interval on ECG
B) Cca 1.8
C) K 3.3
D) Na 110
E) Prolongation of the QTc interval on ECG

A

Digoxin is used for the treatment of chronic heart failure and AF. It inhibits Na+-K- ATPase pump in myocardial cells leading to increased contractility. In hypokalemia, it becomes easier for digoxin to bind to ATPase, leading to toxicity. Furosemide and other diuretics can lower potassium levels, resulting in digoxin toxicity.

Hypernatraemia, hypercalcaemia and hypothyroidism can all precipitate digoxin toxicity.

Digoxin toxicity may be associated with the prolongation of the PR interval, but it does not indicate the cause of the toxicity

How well did you know this?
1
Not at all
2
3
4
5
Perfectly
97
Q

Which one of the following features is least recognised in long-term lithium use?
A) Alopecia
B) Diarrhoea
C) Fine tremor
D) Goitre
E) Diarrhoea

A

All the above side-effects, with the exception of alopecia, may be seen in patients taking lithium

How well did you know this?
1
Not at all
2
3
4
5
Perfectly
98
Q

A 45F presents with a 2M history of left-hand weakness. O/E, there is mild weakness of the L upper and lower limbs with a R-sided facial weakness which spares the forehead. Where is the lesion?
A) Right cerebrum
B) Left cerebrum
C) Right pons
D) Left pons
E) Cervical spinal cord

A

The pons is above the level of decussation of the corticospinal tracts so a pontine lesion would cause a contralateral limb weakness. The facial motor nucleus is located in the pons and supplies the ipsilateral facial muscles. Therefore the answer is the right pons.

How well did you know this?
1
Not at all
2
3
4
5
Perfectly
99
Q

A patient presents sudden onset weakness of her right lower leg while preparing breakfast. O/E she also has decreased touch sensation on the same side. She has a PMHx of hypercholesterolaemia, hypertension and T2DM. CT scan showed an area of hypo-attenuation in the brain. Which artery is the most likely to be occluded in this patient?
A) Posterior cerebral artery
B) Middle cerebral artery
C) Anterior cerebral artery
D) Basilar artery
E) PICA

A

Contralateral hemiparesis and sensory loss with the lower extremity being more affected than the upper = anterior cerebral artery

How well did you know this?
1
Not at all
2
3
4
5
Perfectly
100
Q

Which of the following would likely worsen the prognosis of CLL?
A) Increased serum beta-2-microglobulin
B) Being female
C) Developing CLL at younger age
D) Deletion of 17p13 region
E) Deletion of 13q14 region

A

CLL prognosis can vary from 2 to 20 years. Fluorescence in situ hybridization tests (FISH) panel tests, flow cytometry and lab markers are used in order to gather information on prognosis.

Deletion of 17p13 is the strongest independent prognostic factor for CLL. The disease progresses more rapidly and tends to be refractory to many conventional treatments.

The most common genetic aberration in CLL is deletion of 13q. It is associated with a more indolent course of the disease improving the prognosis.

How well did you know this?
1
Not at all
2
3
4
5
Perfectly
101
Q

A 55M presents complaining of visual disturbance. Examination reveals a right congruous homonymous hemianopia with macula sparing. Where is the lesion most likely to be?
A) R optic nerve
B) R optic radiation
C) L optic tract
D) L occipital cortex
E) Optic chiasm

A

Incongruous defects = optic tract lesion; congruous defects = optic radiation lesion or occipital cortex

A congruous defect means complete or symmetrical visual field loss and conversely an incongruous defect is incomplete or asymmetric.

Homonymous hemianopia
-incongruous defects: lesion of optic tract
-congruous defects: lesion of optic radiation or occipital cortex
-macula sparing: lesion of occipital cortex

How well did you know this?
1
Not at all
2
3
4
5
Perfectly
102
Q

A 45M is on maximum dose ramipril for HTN but is still found to be hypertensive. What medication should be added?
A) Bisoprolol
B) Doxazosin
C) Indapamide
D) Spironolactone
E) Verapamil

A

Indapamide

Verapamil is a rate-limiting CCB and is not used for the management of hypertension.

How well did you know this?
1
Not at all
2
3
4
5
Perfectly
103
Q

A patient is found to have mild lymphocytosis and their Toxoplasmosis serology is positive. What would be the most appropriate next step?
A) Amphotericin
B) Metronidazole
C) Pyrimethamine
D) Spiramycin
E) Supportive care

A

Supportive care with fluids and analgesia -most infections caused by Toxoplasma gondii are self-limiting, and treatment is not necessary for immunocompetent people. Exceptions include patients with eye involvement (toxoplasma chorioretinitis) and those who are immunosuppressed (AIDS).

If treatment is required (e.g. toxoplasmic encephalitis related to AIDS) then a combination of pyrimethamine and sulfadiazine is usually given for several weeks.

How well did you know this?
1
Not at all
2
3
4
5
Perfectly
104
Q

A 72M has exertional chest pain and a positive exercise tolerance test. He is discharged on a combination of aspirin 75mg od, simvastatin 40mg on, atenolol 50mg od and a GTN spray prn. O/E his pulse is 72 bpm and a BP 130/80 mmHg. He is still regularly using his GTN spray. What is the most appropriate next step in management?
A) Add nifedipine MR 30mg OD
B) Add ISMN 30mg BD
C) Increase atenolol to 100mg OD
D) Add nicorandil 10mg BD
E) Add verapamil 80mg TDS

A

When treating angina, if there is a poor response to the first-line drug (e.g. a beta-blocker), the dose should be titrated up before adding another drug. The BNF recommends an atenolol dose of 100mg daily in 1 or 2 doses for angina. The starting dose of isosorbide mononitrate is 10mg bd.

NICE recommend using either a B-blocker or a CCB first-line based on ‘comorbidities, contraindications and the person’s preference’
-if a CCB is used as monotherapy, a rate-limiting one such as verapamil or diltiazem should be used
-if used in combination with a B-blocker then use a longer-acting dihydropyridine CCB (e.g. amlodipine, modified-release nifedipine)
-B-blockers should not be prescribed concurrently with verapamil (risk of complete heart block)

If there is a poor response to initial treatment then medication should be increased to the maximum tolerated dose. If a patient is still symptomatic after monotherapy with a B-blocker add a CCB and vice versa

How well did you know this?
1
Not at all
2
3
4
5
Perfectly
105
Q

Which of the following investigations are first-line for acromegaly?
A) GH levels
B) Faecal elastase
C) MRI pituitary gland
D) OGTT
E) Serum IGF-1 levels

A

Serum IGF-1 levels are now the first-line test for acromegaly. The OGTT test is recommended to confirm the diagnosis if IGF-1 levels are raised.

Growth hormone (GH) levels vary during the day and are therefore not diagnostic.

How well did you know this?
1
Not at all
2
3
4
5
Perfectly
106
Q

Which of the following investigations are first-line for acromegaly?
A) GH levels
B) Faecal elastase
C) MRI pituitary gland
D) OGTT
E) Serum IGF-1 levels

A

Serum IGF-1 levels are now the first-line test for acromegaly. The OGTT test is recommended to confirm the diagnosis if IGF-1 levels are raised.

Growth hormone (GH) levels vary during the day and are therefore not diagnostic.

How well did you know this?
1
Not at all
2
3
4
5
Perfectly
107
Q

Which of the following complications is a patient on anastrozole for breast cancer treatment most likely to develop?
A) Increased likelihood of cancer recurrence
B) Reduced serum HDL levels
C) MI
D) VTE
E) Osteoporotic fracture

A

Aromatase inhibitors (e.g. anastrozole) may cause osteoporosis. Aromatase inhibitors block the synthesis of oestrogen in peripheral tissues, which is the primary source of oestrogen in postmenopausal women. This has a detrimental effect on BMD and increased risk of #. Measurement of BMD is essential before commencing treatment and at regular intervals thereafter, with strong consideration for bisphosphonate prophylaxis.

VTE is also not linked to anastrozole treatment, it is however linked to SERM treatment. This is more of a risk in postmenopausal women who are prescribed oestrogen (rather than oestrogen-depleting drugs), such as those receiving hormone replacement therapy.

How well did you know this?
1
Not at all
2
3
4
5
Perfectly
108
Q

A 55Mpresents with a fever and yellow skin and eyes. He has recently had an illness characterized by fever, chills, headache, and myalgia after returning from Angola. He seemed to recover from the illness and was well for a period of 48 hours before deteriorating. He has a past medical history of rheumatoid arthritis and is on tocilizumab. O/E, there is bleeding from his eyes and mucus membranes and he is visibly jaundiced. The temperature is 38.5ºC. What is the likely diagnosis?
A) Ebola
B) Lassa fever
C) Malaria
D) Typhoid
E) Yellow fever

A

The patient presents with a flu-like illness after returning from Nigeria, followed by a brief period of remission with subsequent recrudescence of fever and the development of jaundice and haemorrhage from the eyes and mucus membranes. This presentation is typical of yellow fever (and the brief period of remission is characteristic). Tocilizumab treatment for rheumatoid arthritis means the yellow fever vaccine is contraindicated and would put the patient at increased risk of developing the illness.

How well did you know this?
1
Not at all
2
3
4
5
Perfectly
109
Q

Which of the following RF may pre-dispose a patient to Charles-Bonnet syndrome?
A) Caucasian
B) Male
C) Occupational history of working in sewers
D) Peripheral visual impairment
E) Hypertension

A

Charles-Bonnet syndrome is characterised by visual hallucinations associated with eye disease. Peripheral visual impairment is a risk factor.

How well did you know this?
1
Not at all
2
3
4
5
Perfectly
110
Q

In nephrogenic diabetes insipidus, what is the most likely mutation?
A) Aquaporin 2 channels
B) Epithelial sodium channels
C) Na/Cl pumps
D) Na/K/Cl pumps
E) Vasopressin receptors

A

Nephrogenic diabetes insipidus may be caused genetic mutations:
-the more common form affects the vasopression (ADH) receptor
-the less common form results from a mutation in the gene that encodes the aquaporin 2 channel

In nephrogenic DI, the kidney is unable to respond to circulating antidiuretic hormone (ADH) from the posterior pituitary, causing an inability to reabsorb water within the collecting tubules.

How well did you know this?
1
Not at all
2
3
4
5
Perfectly
111
Q

Which one of the following immunoglobulins is involved in the activation of B-cells?
A) IgD
B) IgM
C) IgG
D) IgA
E) IgE

A

IgD is involved in the activation of B-cells.

How well did you know this?
1
Not at all
2
3
4
5
Perfectly
112
Q

In a patient with metastatic colon cancer, what organism is most likely to cause IE?
A) Streptococcus gallolyticus
B) Brucella
C) Coxiella burnetti
D) Staphylococcus epidermidis
E) Streptococcus viridans

A

Streptococcus bovis
-associated with colorectal cancer
-the subtype Streptococcus gallolyticus is most linked with colorectal cancer

How well did you know this?
1
Not at all
2
3
4
5
Perfectly
113
Q

Which of the following investigation findings would support a diagnosis of idiopathic pulmonary fibrosis?
A) Reticular changes on CT worse at the bases
B) Obstructive picture on spirometry
C) Extensive ground glass opacities on CT
D) Increased transfer factor on spirometry
E) Lymphocytosis on BAL

A

On CT imaging the changes are often seen are honeycombing, reticular opacities, traction bronchiectasis, and architectural distortion. Ground-glass opacities are less of a feature of idiopathic pulmonary fibrosis. The changes are usually more pronounced at the bases.

How well did you know this?
1
Not at all
2
3
4
5
Perfectly
114
Q

What part of the nephron is the majority of glucose reabsorbed?
A) Glomerulus
B) DCT
C) PCT
D) Descending loop of Henle
E) Ascending loop of Henle

A

Glucose is usually almost fully reabsorbed in the proximal convoluted tubules (PCT) by secondary active, sodium-coupled transporters (SGLT2).

How well did you know this?
1
Not at all
2
3
4
5
Perfectly
115
Q

Looser’s zones x-ray are most characteristically associated with which one of the following conditions?
A) Hypoparathyroidism
B) Primary hyperparathyroidism
C) Osteomalacia
D) Pagets disease
E) Osteoporosis

A

Osteomalacia

How well did you know this?
1
Not at all
2
3
4
5
Perfectly
116
Q

In a patient with CF, chronic infection with what organism would be a contraindication to lung transplant?
A) Aspergillus fumigatus
B) Burkholderia cepacia
C) Mycobacterium abscessus
D) Pseudomonas aeruginosa
E) Scedosporium apiospermum

A

Chronic infection with Burkholderia cepacia is an important CF-specific contraindication to lung transplantation.

How well did you know this?
1
Not at all
2
3
4
5
Perfectly
117
Q

How should a patient who is unsure of their vaccination status be managed if they received a bite from a bat?
A) Prophylactic antibiotics
B) Rabies Ig
C) Rabies vaccination and Ig
D) Reassurance
E) Tetanus Ig

A

Rabies is the particular worry after a bat bite. High-risk non-immune patients should be treated with the rabies vaccination and immunoglobulin. If the patient has previously been vaccinated, then only a repeat course of the vaccination is needed with no need to administer immunoglobulin.

How well did you know this?
1
Not at all
2
3
4
5
Perfectly
118
Q

A 25M presents with multiple painful penile ulcers with ragged edges and inguinal lymphadenopathy. He admits to unprotected sexual intercourse. Gram-stain of ulcer swabs shown gram-negative rods. Which of the following is the most likely diagnosis?
A) Chancroid
B) Syphilis
C) HSV
D) Chlamydia
E) Lymphogranuloma venereum

A

Chancroid is caused by Haemophilus ducreyi and presents as a tender ulcer with a ragged edge that readily bleeds on contact. Multiple ulcers can occur and there is often associated lymphadenopathy.

LGV is incorrect due to the painful nature of the ulcers and Gram-ve rods. LGV is caused by Chlamydia trachomatis.

How well did you know this?
1
Not at all
2
3
4
5
Perfectly
119
Q

Which of the following regular medications would you prescribe to a patient with an egg-allergy that causes them to break out in hives?
A) Ranitidine
B) Prednisolone
C) Topical hydrocortisone
D) Chlorpheniramine
E) Topical lidocaine

A

Chlorpheniramine can be used to reduce the duration and severity of hives.

How well did you know this?
1
Not at all
2
3
4
5
Perfectly
120
Q

Which of the following is recognised as as a main adverse effect of steroid therapy?
A) Hypersomnia
B) Increased gluconeogenesis
C) Increased osteoblast activity
D) Increased renal sodium loss
E) Peripheral adiposity

A

Corticosteroids lead to insulin resistance, one consequence of which is increased gluconeogenesis.

It is central adiposity that is increased by steroid therapy.

121
Q

Which organ is in direct contact with the anterior surface of the L kidney (and not separated by peritoneum)?
A) Duodenum
B) Jejunum
C) Pancreas
D) Spleen
E) Stomach

A

Only the pancreas and the L suprarenal gland make direct contact with the L kidney, with no intervening peritoneum.

122
Q

Which of the following medications would be most appropriate in the management of an acute cluster headache?
A) Aspirin
B) Atenolol
C) Levocabastine
D) Paracetamol
E) Sumatriptan

A

Sumatriptan is the medication of choice in an acute attack. Verapamil or lithium can be used in prevention if attacks are frequent and last >3W.

123
Q

What is the MOA of sitagliptin?
A) DPPIV inhibitor
B) AMP kinase activator
C) Glucokinase activator
D) SGLT2 inhibitor
E) GLP-1 agonist

A

DPPIV inhibitor. GLP-1 is leaved by DPPIV - hence its inhibition will prolong the 1/2 life of GLP-1, leading to an improvement in insulin release.

Liraglutide is a GLP-1 agonist.

124
Q

A 78M with chronic venous insufficiency presents with a very well-demarcated area of intense erythema on the L leg. There is induration and a raised border. What is the most likely causative organism?
A) Group A b-haemolytic Strpetococcus
B) H. influenzae
C) S. Aureus
D) P. aeruginosa
E) S. viridian’s

A

This is a typical presentation of erysipelas - a superficial form of cellulitis caused by Group A b-haemolytic Streptococcus.

125
Q

What is the usual pattern of inheritance for Wiskott-Aldrich syndrome?
A) AR
B) AD
C) X-linked R
D) X-linked D
E) Y-linked

A

WAS is inherited in an X-linked recessive pattern, making male children more prone to the disease.

125
Q

A patient presents with a rapidly enlarging thyroid mass, and worsening stridor. O/E there is a large hard thyroid mass, largest over the L hand side of the gland but extending to the R with regional lymphadenopathy. What is the most likely diagnosis?
A) Anapaestic thyroid cancer
B) Follicular thyroid cancer
C) Medullary thyroid cancer
D) Papillary thyroid cancer
E) Thyroid lymphoma

A

Anaplastic thyroid carcinoma is most consistent with the rapid spread of this malignancy. US-guidedFNA is the usual method of diagnosis. Survival is very poor.

126
Q

Which of the following medications is indicated for remission in a child suffering with recurrent episodes of minimal change disease managed by steroids?
A) ACEi
B) Cyclosporine A
C) Cyclophosphamide
D) Nephrectomy
E) Prednisolone 60mg/day

A

Frequent relapses managed with high dose steroids should prompt treatment with alkylating agents (cyclophosphamide).

ACEi can be used to manage the condition but will not induce remission. Nephrectomy is only a last resort.

127
Q

A 30M under investigation for abdominal cramps and passing blood rectally presents with an acutely red, painful, photophobic eye. What is the most likely sign on ocular examination?A) Conjunctival purulent discharge
B) White corneal stroll infiltrate
C) Mydriasis of affected eye
D) Hipopyon
E) Swollen optic disc

A

This man is likely to be suffering with IBD, and hence in this context, iritis must be considered. The classical symptom of iritis is photophobia. A cardinal sign of iritis is the presence of inflammatory cells in the anterior chamber, visible with a slit lamp. In severe iritis, the inflammation is severe enough for the inflammatory cells inferiorly to form a level - a hypopyon.

128
Q

A 50F presents with stridor and noisy inspiratory effort and difficulty breathing following a laparoscopic cholecystectomy. She is on long-term warfarin for thromboembolic disease, salbutamol and inhaled steroids for asthma and penicillamine for severe rheumatoid arthritis. Which of the following tests would be most useful in determining the cause of her post-op SOB?
A) CXR
B) CTPA
C) Peak flow
D) Spirometry with flow-volume loop
E) Spirometry with transfer factor measurement

A

This patient has stridor due to cricoarytenoid arthritis which is seen in 75% of patients with RA. It can cause stridor, sore throat and hoarse voice, but it is often asymptomatic. However, symptoms can rapidly worsen in the post-operative period. Spirometry with flow-volume loop is the diagnostic method. Patient may need urgent tracheostomy and steroids.

129
Q

With respect to the pharmacokinetics of digoxin, which characteristic drives the requirement for a loading dose?
A) Bioavailability
B) First pass metabolism
C) Half-life
D) Protein binding
E) Volume of distribution

A

Digoxin has a large volume of distribution and penetrates tissues including the heart, brain, erythrocytes and skeletal muscle.

130
Q

Which of the following vectors is most likely responsible for the spread of epidemic typhus?
A) Aedes aegypti mosquito
B) Hard tick
C) Human body louse
D) Rat flea
E) Trombiculid mite

A

The chief vector of epidemic typhus is the human body louse. Once infected, Rickettsia prowazekii secretes the organism in its faeces which are then rubbed into the bite wound.

131
Q

pWhich IV antibiotic regime is most appropriate in treating a liver abscess in a penicillin allergic patient (diarrhoea)?
A) Azithromycin + clindamycin
B) Ceftriaxone + metronidazole
C) Clindamycin + ciprofloxacin
D) Clindamycin + metronidazole
E) Vancomycin + meropenem

A

The most frequently implicated organisms in liver abscesses are Gram-ve bacteria, Gram+ anaerobes and other anaerobic organisms. Ideally treatment should include a penicillin based b-lactam antibiotic combined with metronidazole. Where a penicillin allergy is unlikely/non-severe, a cephalosporin can be substituted in. Hence ceftriaxone + metronidazole is the correct answer.

132
Q

Which of the following drugs reduce the efficacy of adenosine?
A) Dipyridamole
B) Bupivacaine
C) Aminophylline
D) Amiodarone
E) Montelukast

A

Aminophylline reduces the effect of adenosine.

The effects of adenosine are enhanced by dipyridamole (antiplatelet agent) and blocked by theophyllines. It should be avoided in asthmatics due to possible bronchospasm.

133
Q

What is the most appropriate initial investigation for a patient presenting with haemachromatosis?
A) Transferrin saturation + ferritin
B) Haematocrit + ferritin
C) Liver biopsy with Perl’s stain
D) Serum iron + ferritin
E) Serum iron + haematocrit

A

Measuring the transferrin saturation first-line as this is the most specific and sensitive test for iron accumulation. Serum ferritin should also be measured but this marker is not usually abnormal in the early stages of iron accumulation. Family members can also be tested for HFE gene.

134
Q

What is the mechanism of action of sumatriptan?
A) 5-HT1 agonist
B) 5-HT1 antagonist
C) 5-HT2 agonist
D) 5-HT2 antagonist
E) 5-HT3 agonist

A

Triptans are serotonin 5-HT1B and 5-HT1D receptor agonists.

135
Q

Which monoclonal antibody can be used in recurrent C. difficile infection?
A) Bezlotoxumab
B) Obiltoxaximab
C) Raxibacumab
D) Suvratoxumab
E) Urtoxazumab

A

Bezlotoxumab is a monoclonal antibody which binds to Clostridium difficile toxin B and neutralises it to prevent recurrence of infection.

136
Q

Which one of the following translocations is associated with acute promyelocytic leukaemia?
A) t(15;17)
B) t(9;17)
C) t(9;22)
D) t(15;22)
E) t(17;22)

A

Acute promyelocytic leukaemia - t(15;17)

Features
-presents younger than other types of AML (average = 25 years old)
-DIC or thrombocytopenia often at presentation
-good prognosis

137
Q

Which of the following medications may precipitate acute glaucoma?
A) Methotrexate
B) Doxazosin
C) Amitriptyline
D) Atenolol
E) Bendroflumethiazide

A

Drugs which may precipitate acute glaucoma include anticholinergics and tricyclic antidepressants (amitriptyline)

138
Q

Which of the following factors are not associated with pulmonary artery hypertension?
A) Eisenmenger syndrome
B) HIV
C) Hepatitis B
D) Sickle cell anaemia
E) Sarcoidosis

A

Hepatitis B

139
Q

What is the treatment of Cryptosporidium diarrhoea in a HIV+ve patient?
A) Ivermectin
B) Mebendazole
C) Ciprofloxacin
D) Metronidazole
E) Nitazoxanide

A

Nitazoxanide is a medication that is used to treat diarrhoea caused by certain parasite infections of the intestines like from Cryptosporidium parvum. It is a broad-spectrum thiazolide anti-parasitic agent.

140
Q

What is the single most important piece of advice to give a patient who had just recovered from a pneumothorax to prevent their risk of a further pneumothorax?
A) Avoid flying for 12M
B) Avoid contact sports for 12M
C) Stop smoking
D) Arrange a course of respiratory physiotherapy
E) Seek prompt medical advice for respiratory infections

A

All patients should be advised to avoid smoking to reduce the risk of further episodes - the lifetime risk of developing a pneumothorax in healthy smoking men is around 10% compared with around 0.1% in non-smoking men.

‘Fitness to fly’ rules the CAA suggest patients may travel 2 weeks after successful drainage if there is no residual air. BTS used to recommend not travelling by air for a period of 6 weeks but this has now been changed to 1 week post check x-ray. Diving should be permanently avoided unless the patient has undergone bilateral surgical pleurectomy and has normal lung function and chest CT scan postoperatively.

141
Q

Which of the following is not associated with oesophageal cancer?
A) H. pylori infection
B) Achalasia
C) GORD
D) Alcohol
E) Smoking

A

Helicobacter pylori may actually be protective against oesophageal cancer.

141
Q

What is the recommended antibiotic treatment in a patient with a prosthetic valve and staphylococcal endocarditis?
A) Vancomycin + rifampicin
B) Flucloxacillin + rifampicin + low-dose gentamicin
C) Benzypenicillin
D) Benzypenicillin + low-dose gentamicin
E) Vancomycin + rifampicin + low-dose gentamicin

A

Flucloxacillin + rifampicin + low-dose gentamicin

Vancomycin + rifampicin + low-dose gentamicin are used in penicillin-allergic patients with prosthetic valve endocarditis caused by staphylococci.

Vancomycin + rifampicin are used in penicillin-allergic patients with native valve endocarditis caused by staphylococci.

142
Q

What is the most appropriate test to confirm eradication of H. pylori?
A) Urea breath test
B) Culture of gastric biopsy
C) H. pylori serology
D) Hydrogen breath test
E) Stool culture

A

Urea breath test is the only test recommended for H. pylori post-eradication therapy - should not be performed within 4 weeks of treatment with an antibacterial or within 2 weeks of an antisecretory drug (e.g. a proton pump inhibitor).

143
Q

What is the mechanism of action of nicorandil?
A) Fast sodium channel antagonist
B) Nitric oxide reductase inhibitor
C) Acts on I(f) ion in SAN
D) Potassium channel activator
E) Glutathione-S-transferase inhibitor

A

Nicorandil is a vasodilatory drug used to treat angina. It is a potassium-channel activator with vasodilation is through activation of guanylyl cyclase which results in increase cGMP.

144
Q

The action of which one of the following brush border enzymes results in the formation of glucose and galactose?
A) Dipeptidase
B) A-dextrinase
C) Maltase
D) Lactase
E) Sucrose

A

Sucrose = glucose + fructose
Maltase = glucose + glucose
Lactase = glucose + galactose

145
Q

A 34M presents with increasing breathlessness over the last few weeks. He complains of increasing wheeze and dark sputum production. His has asthma, which until recently has been well controlled on salbutamol, formoterol and low-dose beclomethasone inhalers. A CXR shows central bronchiectasis and bloods show eosinophilia. What is the most appropriate management?
A) Weaning course of prednisolone
B) 21-day course of co-trimoxazol
C) 4-month course of itraconazole
D) Increasing the dose of beclometasone
E) Short course of oral prednisolone and amoxicillin

A

This patient has a history of asthma and is presenting with symptoms of wheeze, dyspnoea and productive cough. The blood tests show eosinophilia and the chest x-ray shows proximal bronchiectasis. This is all suggestive of allergic bronchopulmonary aspergillosis. The first-line treatment is a course of oral steroids.

146
Q

What are the most common pulmonary manifestations of SLE?
A) Atelectasis
B) Pleural effusion
C) Pneumonitis
D) Pulmonary fibrosis
E) Restrictive lung disease

A

Pleurisy and pleural effusions are the most common pulmonary manifestations of SLE and are often B/L.

147
Q

A 45F with depression is currently on selegilline. Her GP would like to start her on sertraline. What would be the most appropriate prescription?
A) Continue selegilline and start paroxetine
B) Start paroxetine and slowly wean selegilline
C) Stop selegilline and start paroxetine straight away
D) Stop selegilline, wait 14 days, start paroxetine
E) Stop selegilline, wait 6W, start paroxetine

A

Selegilline is a MAOi which also increases the serotonin levels in the brain and hence, if switching to paroxetine, increases the risk of developing serotonin syndrome. Hence a washout period of 2W is necessary before starting the new medication.

148
Q

Which of the following adverse effects is most commonly associated with moxifloxacin?
A) Achilles tendon rupture
B) C diff associated diarrhoea
C) Neutropenia
D) Oesophageal rupture
E) Widening of QRS

A

Moxifloxacin is the new generation of antibiotics that is bactericidal and interferes with DNA synthesis. It is effective against Gram+ bacteria. Adverse effects include the risk of tendonitis and tendon rupture.

149
Q

A patient is due for extraction of teeth under local anaesthesia. He has a past history of rheumatic heart disease and mitral stenosis. Which of the following would be the most appropriate antibiotic prophylaxis for him?
A) Amoxicillin
B) Co-amoxiclav
C) Ciprofloxacin
D) No prophylaxis needed
E) Vancomycin

A

Antibiotic prophylaxis is not recommended for dental extractions - including for patients with rheumatic heart disease or those who have undergone placement.

150
Q

Which of the following statements regarding SLE is true?
A) CRP rise is relatively modest
B) RF is positive in <5%
C) Neutropenia is more common than lymphopenia
D) The COCP and HRT are contraindicated
E) Pulmonary fibrosis is a common disease complication

A

Elevation of ESR without CRP rise is typical of SLE.

Lung fibrosis is relatively uncommon in SLE and is a late feature. Lymphopenia is the typical haematological finding.

151
Q

Which of the following statements regarding hereditary angioedema is correct?
A) Persistently low C3 levels, including between attacks
B) Persistently low C4 levels, including between attacks
C) Search for a paraprotein
D) Drug history of taking ACEi
E) Urticaria with angio-oedematous swellings

A

C4 levels usually remain low between attacks.

152
Q

A 25F presents with recurrent attacks of ‘dizziness’. These attacks typically last around 30-60 minutes and occur every few days or so. During an attack ‘the room seems to be spinning’ and the patient often feels sick. These episodes are often accompanied by a ‘roaring’ sensation in the left ear. Otoscopy is normal but Weber’s test localises to the right ear. What is the most likely diagnosis?
A) Acoustic neuroma
B) Vestibular neuritis
C) BPPV
D) MS
E) Meniere’s disease

A

Meniere’s disease is a disorder of the inner ear of unknown cause; characterised by excessive pressure and progressive dilation of the endolymphatic system.

Features
-recurrent episodes of vertigo, tinnitus and sensorineural hearing loss. Vertigo is usually the prominent symptom
-a sensation of aural fullness or pressure
-nystagmus
-positive Romberg test

Episodes last minutes to hours; typically symptoms are unilateral but bilateral symptoms may develop

ENT assessment is required to confirm the diagnosis - patients should inform the DVLA. The current advice is to cease driving until satisfactory control of symptoms is achieved.

153
Q

Which one of the following statements regarding relative risk is correct?
A) Relative risk = 1-ARR
B) It is the usual outcome measure of cohort studies
C) Risk may be defined as the odds of an outcome happening
D) Relative risk = 1/odds ratio
E) If the risk ratio is <1 then the risk of an event is increased compared to controls

A

It is the usual outcome measure of cohort studies.

Risk and odds are different. If 20 patients die out of every 100 who have a myocardial infarction then the risk of dying is 20 / 100 = 0.2 whereas the odds are 20 / 80 = 0.25.

154
Q

What is the mechanism of action of pegvisomant (for acromegaly)?
A) IGF-1 receptor antagonist
B) GH receptor antagonist
C) IGF-1 receptor antagonist
D) GH receptor agonist
E) Somatostatin analogue

A

Trans-sphenoidal surgery is the first-line treatment unless the tumour is inoperable or surgery unsuccessful - hence medication may be needed.

somatostatin analogues directly inhibit the release of GH – octreotide

GH receptor antagonist prevents dimerization of the GH receptor – pegvisomant – very effective but doesn’t reduce tumour volume (surgery still needed if mass effect)

dopamine agonists (superseded by octreotide) – bromocriptine

155
Q

Which one of the following features is characteristic of acute intermittent porphyria?
A) Photosensitivity
B) Increased urinary porphobilinogen between attacks
C) Hypernatraemia during attacks
D) AR inheritance
E) Increased faecal protoporphyrin excretion

A

AIP is a rare AD condition caused by a defect in porphobilinogen deaminase (enzyme involved in haem biosynthesis). It characteristically presents with abdominal and neuropsychiatric symptoms in 20-40-year-olds. AIP is more common in females (5:1).

Diagnosis
-urine turns deep red on standing
-raised urinary porphobilinogen (elevated between attacks and to a greater extent during acute attacks)
-assay of red cells for porphobilinogen deaminase
-raised serum levels of delta aminolaevulinic acid and porphobilinogen

156
Q

Which one of the following markers is most useful in assessing risk of sudden death in a patient with HOCM?
A) Abnormal BP changes on excercise
B) Left ventricular outflow tract gradient
C) QT interval
D) Right atrial diameter
E) QRS length

A

Poor prognostic factors
-syncope
-family history of sudden death
-young age at presentation
-non-sustained ventricular tachycardia on 24 or 48-hour Holter monitoring
-abnormal blood pressure changes on exercise

157
Q

Following a stroke, a patient is unable to get words out easily and his speech is non-fluent as if hesitating before uttering the words. During the CN examination, he understood and followed your instructions well. However, he is unable to repeat words after you. What type of speech disturbance does this patient have?
A) Broca’s aphasia
B) Conductive aphasia
C) Wernicke’s aphasia
D) Global aphasia
E) Dysarthria

A

Patients with damaged Broca’s area develop expressive dysphasia. They struggle to get the words out of their mouth despite them knowing what they want to say. They do not have a problem understanding and following instructions but cannot repeat words easily despite understanding them.

Conductive dysphasia presents with fluent speech but an inability to repeat words. This patient had trouble formulating words and his speech was not fluent.

158
Q

A farmer presents with headache, fever and muscle aches with his symptoms getting progressively worse over the past week. He reports nausea and a decreased urine output. O/E: T 38.2ºC, HR 102bpm, chest clear. Subconjunctival haemorrhages are noted but there is no evidence of jaundice. What is the most likely diagnosis?
A) Lyme disease
B) Mycoplasma pneumonia
C) Legionella pneumonia
D) Listeria
E) Leptospirosis

A

Leptospirosis is classically spread by contact with infected rat urine - commonly seen in sewage workers, farmers, vets or people who work in an abattoir

Features
1. early phase is due to bacteraemia and lasts approx 1 week
-may be mild or subclinical
-fever
-flu-like symptoms
-subconjunctival suffusion (haemorrhage)
2. immune phase may lead to more severe disease (Weil’s disease)
-AKI
-hepatitis: jaundice, hepatomegaly
-aseptic meningitis

159
Q

Which of the following medications exhibit zero-order kinetics?
A) Aspirin
B) Bisoprolol
C) ISMN
D) Ivabradine
E) Ramipril

A

Drugs which exhibit zero-order kinetics include phenytoin, alcohol and salicylates (aspirin).

Zero-order kinetics refers to the rate of excretion being constant, regardless of changes in plasma concentration of the drug. This is due to saturation of the metabolic process underlying the excretion.

Most medications exhibit first-order kinetics where the rate of drug elimination is proportional to drug concentration.

160
Q

A 54F with known breast cancer develops progressive motor disorder with impairment of right arm function. She is clumsy with all movements and exaggerated ‘flinging’ actions are noted on examination. There is a similar pattern in the right leg with to a much lesser extent. There are no other abnormalities on neurological examination. Where is the lesion most likely to be?
A) Caudate nucleus
B) Lateral cerebellum
C) Pre-motor cortex
D) Sub-thalamic nucleus
E) Temporal lobe

A

Hemiballismus is a movement disorder characterised by very violent movements of the arm or limbs. Ballisic movements are involuntary, sudden, jerking movements which occur contralateral to the side of the lesion. They primarily affect the proximal limb musculature whilst the distal muscles may display more choreiform-like movements Caused by a stroke or other lesion of the subthalamic nucleus.

Movement disorders in order of least speed to fastest (DACB)
Dystonia - fixated position
Athetosis - Snake-like writhing (slow)
Choreiform - Like a dance choreographer
Ballistic/Ballismus/Hemiballismus - Fast flinging movements, can injure themselves or others ‘like a ballistic missile’ (memorisation method)

161
Q

Which TB medication needs to be adjusted in renal failure?
A) Isoniazid
B) Rifampicin
C) Pyrazinamide
D) Ethambutol
E) No changes

A

Ethambutol S/E
-optic neuritis: check visual acuity before and during treatment
-dose needs adjusting in patients with renal impairment

162
Q

Which of the following conditions is not inherited in an X-linked recessive fashion?
A) Myotonic dystrophy
B) Fabry’s disease
C) G6PD
D) Colour blindness
E) Haemophilia B

A

Myotonic dystrophy

X-linked conditions: Duchenne/Becker, haemophilia, G6PD

163
Q

A 34F with poorly controlled HIV present with progressive weakness in her lower limbs. She also states that she is finding it difficult to walk up the stairs and is becoming generally clumsy. She had no other systemic symptoms and no other PMHx. O/E, power is normal in UL and LL B/L but there is B/L dysmetria. She had an MRI which showed some white matter lesions bilaterally in the parietal lobes. What is the most likely diagnosis?
A) Cerebral vasculitis
B) Progressive multifocal leukoencephalopathy
C) MS
D) Toxoplasmosis
E) Brain metastases

A

Multiple sclerosis and PML lead to white matter lesions on MRI. However, with her background of HIV and being non-compliant with her medication, she is at risk of neuro-complications from being immunocompromised. Therefore PML is the more likely diagnosis.

164
Q

A 35F presents with polyarthralgia affecting her hands and feet. There is no evidence of synovitis but her fingers and toes are noted to be dusky and blue, without evidence of ulceration. There is proximal pain and tenderness with power 4+/5 at the shoulders and hips bilaterally. Chest auscultation reveals bibasal end-inspiratory crackles, which do not clear on coughing. Blood tests show a raised CK, normal complement levels and positive for anti-ribonuclear protein antibodies. What is the most likely diagnosis?
A) Dermatomyositis
B) Mixed connective tissue disease
C) Scleroderma
D) Seronegative RA
E) SLE

A

MCTD is a connective tissue disease that has classically been described as having features of multiple connective tissue diseases (e.g. myositis, scleroderma, sjogren’s, SLE) and an association with anti-ribonuclear protein (anti-RNP) antibody.

165
Q

What is the most appropriate method of assessing for publication bias in a meta-analysis?
A) Forrest plot
B) Funnel plot
C) GRADE analysis
D) Meta-regression analysis
E) Sensitivity analysis

A

A funnel plot is a type of scatter plot used in meta-analyses to detect publication bias visually.

A forest plot is a graphical presentation of a meta-analysis.

Meta-regression is an extension of subgroup meta-analysis that allows the effects of continuous and/or categorical characteristics to be investigated.

166
Q

Which one of the following would suggest a diagnosis of Parkinson’s disease rather than drug-induced parkinsonism?
A) Rigidity
B) Masked face
C) B/L symptoms
D) Flexed posture
E) Restless arms and legs

A

Rigidity and rest tremor are uncommon in drug-induced parkinsonism.

Masked face and flexed posture can be seen in both conditions. Bilateral symptoms are more common in drug-induced parkinsonism. Restlessness of arms and legs (akathisia) is a common side-effect of antipsychotics.

167
Q

A 67M has a past history of TB as a child but is otherwise normally fit and well. Over the past 2M he has had a cough, lost one stone in weight and had four episodes of haemoptysis. A CXR shows a solid mass occupying the right upper zone and he is found to be aspergillus precipitin antibody +ve. What is the most likely diagnosis?
A) Invasive aspergillosis
B) Asbergilloma
C) ABPA
D) Lung abscess
E) Reactivation of primary TB

A

An aspergilloma is a mycetoma (mass-like fungus ball) which often colonises an existing lung cavity (e.g. secondary to tuberculosis, lung cancer or cystic fibrosis). It is usually asymptomatic but features may include
cough and haemoptysis.

Investigations
-CXR containing a rounded opacity. A crescent sign may be present
-High titres Aspergillus precipitins

168
Q

A 34M presents with a witnessed tonic-clonic seizure on a BG of 3D intermittent fevers and 2W of progressive headache. He has recently had a sinus infection but is normally fit and well. An urgent CT head shows a frontal ring enhancing lesion. What is the most appropriate first-line treatment for this patient?
A) Amphotericin B and fluconazole
B) Ceftriaxone and aciclovir
C) Ceftriaxone and metronidazole
D) Dexamethasone
E) Sulfadiazine and pyrimethamine

A

Given the history and characteristic CT findings, the most likely diagnosis here is a bacterial brain abscess secondary to the patient’s recent sinusitis. The first-line treatment for brain abscess is with broad spectrum antibiotics such as ceftriaxone and metronidazole.

Ceftriaxone is a third-generation cephalosporin with excellent blood brain barrier penetration; it provides good gram-positive and excellent gram-negative cover but lacks anaerobic cover hence the need for the addition of metronidazole (intracranial abscesses secondary to sinusitis are often polymicrobial).

169
Q

What is the most appropriate management of B12 deficiency?
A) 1mg IM hydroxycobalamin every 3M
B) 1 mg of IM hydroxocobalamin 3 times each week for 2 weeks, then once every 3 months
C) 1 mg of IM hydroxocobalamin every 2M
D) 1 mg of IM hydroxocobalamin 3 times each week for 2 weeks
E) 1 mg of IM hydroxocobalamin and the recheck bloods

A

If no neurological involvement 1 mg of IM hydroxocobalamin 3 times each week for 2 weeks, then once every 3 months.

If a patient is also deficient in folic acid then it is important to treat the B12 deficiency first to avoid precipitating subacute combined degeneration of the cord.

170
Q

Which cells develop the antibody responsibe in anaphylactic reactions?
A) Eosinophils
B) Granulocytes
C) Plasma cells
D) Mast cells
E) T-lymphocytes

A

Anaphylaxis is an example of a type I hypersensitivity reaction. During this process, initial exposure to an antigen promotes the synthesis of IgE against the antigen by plasma cells (hence this answer). This IgE then binds to Fc receptors on mast cells. Upon re-exposure, the antigen binds to this surface-bound IgE, triggering mast cell degranulation and an anaphylactic reaction.

171
Q

A 34-year-old intravenous drug user is admitted with a purpuric rash affecting her legs. She is found to be HepC+ve, RF+ve with low C3 and C4. What is the most likely diagnosis?
A) HSP
B) Polyarteritis nodosa
C) GPA
D) SLE
E) Cryoglobulinaemia

A

Hepatitis C infection is associated with type II (mixed) cryoglobulinaemia, suggested by the purpuric rash, positive rheumatoid factor and reduced complement levels.

172
Q

A 24F who has had unprotected sex with multiple partners presents due to recurrent miscarriages, arthralgia and malaise. Treponemal serological tests are negative but the Venereal Disease Research Laboratory/ Rapid Plasma Reagin (VDRL/RPR) test is positive. Which of the following conditions may lead to a false positive test?
A) Rheumatoid arthritis
B) APLS
C) Lyme disease
D) Sarcoidosis
E) Beryllium poisoning

A

Causes of false positive non-treponemal (cardiolipin) tests:
-pregnancy
-SLE, anti-phospholipid syndrome
-tuberculosis
-leprosy
-malaria
-HIV

Non-treponemal tests = RPR, VDRL
Treponemal tests = TP-EIA

Examples of test results
-Positive non-treponemal test + positive treponemal test = active syphilis infection

-Positive non-treponemal test + negative treponemal test = consistent with a false-positive syphilis result

-Negative non-treponemal test + positive treponemal test = consistent with successfully treated syphilis

173
Q

For a patient with a history of HTN on ramipril, what is the cut-off BMI that would trigger a referral for consideration of bariatric surgery?
A) >35
B) >40
C) >30
D) >38
E) >40

A

Obesity - NICE bariatric referral cut-offs
-with risk factors (T2DM, BP etc): > 35 kg/m^2
-no risk factors: > 40 kg/m^2

174
Q

Which of the following ECG findings would be the most consistent with severe hypokalaemia?
A) U waves, T wave inversion, PR shortening
B) U waves, T wave inversion, ST depression
C) U waves, PR shortening, ST depression
D) PR shortening, ST depression, T wave inversion
E) PR lengthening, ST elevation, T wave inversion

A

In hypokalaemia, the quoted figure is that ECG abnormalities begin to appear when K+ falls to below 2.7mmol/L.

ECG features of hypokalaemia
-U waves
-small or absent T waves (occasionally inversion)
-prolong PR interval
-ST depression
-long QT

175
Q

Which of the following investigations is the most appropriate to help stage oesophageal cancer following a CT scan which showed no metastases?
A) Endoscopic US
B) HRCT
C) MRI
D) OGD
E) PET scan

A

If whole-body imaging has not shown any evidence of metastatic disease, EUS is the best investigation for tumour staging and can investigate for mural disease.

OGD is the first-line for assessment of suspected oesophageal and gastric cancers, however is not able to assess for mural invasion and therefore is not the correct answer.

176
Q

Which one of the following medications is contraindicated in heart failure.
A) Sitagliptin
B) Pioglitazone
C) Gliclazide
D) Exenatide
E) Metformin

A

Pioglitazone - contraindicated by: heart failure due to risk of fluid retention.

177
Q

A sample of tissue from a renal biopsy is viewed using an electron microscope. Podocyte fusion is seen. Which one of the following types of glomerulonephritis is most associated with this finding?
A) Membranous glomerulonephritis
B) IgA nephropathy
C) FSGS
D) Mesangiocapillary glomerulonephritis
E) Minimal change glomerulonephritis

A

Podocyte fusion is seen in minimal change glomerulonephritis under electron microscopy but may occasionally be a feature of focal segmental glomerulosclerosis as well. Minimal change glomerulonephritis however is far more common. Minimal change disease is seen as normal glomeruli on light microscopy.

178
Q

Which parts of the spinal cord are affected in subacute combined degeneration of the cord?
A) Anterior and lateral corticospinal tracts
B) Spinocerebellar tract and dorsal column
C) Spinothalamic tract and dorsal column
D) Spinothalamic tract and lateral corticospinal
E) Dorsal columns and lateral corticospinal tracts

A

In subacute combined degeneration of the spinal cord, the dorsal columns, lateral corticospinal tracts and spinocerebellar tracts are affected.

Dorsal column involvement
-distal tingling/burning/sensory loss is symmetrical and tends to affect the legs more than the arms
-impaired proprioception and vibration sense

Lateral corticospinal tract involvement
-muscle weakness, hyperreflexia, and spasticity
-upper motor neuron signs typically develop in the legs first
-brisk knee reflexes
-absent ankle jerks
-extensor plantars

Spinocerebellar tract involvement
sensory ataxia → gait abnormalities
-positive Romberg’s sign

179
Q

What is the best management of osteoporosis in a patient with a normal calcium and a past medical history of VTE and GORD treated with PPI?
A) Calcium and vitamin D
B) Denosumab
C) HRT
D) Raloxifene
E) Risedronate

A

Denosumab is given as 6-monthly SC injections. In a patient with GORD treated with PPI, risedronate runs the risk of significantly worsening oesophagitis.

179
Q

Which of the following best supports a diagnosis of reactive arthritis?
A) Arthritis affecting the UL
B) FHx of UC
C) History of flu-like symptoms 4-6W prior
D) +ve gonococcal culture of urethral discharge
E) Presence of keratoderma blenorrhagica

A

Keratoderma blenorrhagica (brown aseptic abscesses on soles and palms) are present in reactive arthritis.

Reactive arthritis often occurs following gastrointestinal and genitrourinary infections.

180
Q

Which of the following histopathological subtypes of Hodgkins disease most suggests a poor prognosis?
A) Lymphocyte depleted
B) Lymphocyte predominant
C) Mixed cellularity
D) Nodular sclerosis type I
E) Nodular sclerosis type II

A

Hodgkins lymphoma is categorised into nodular lymphocyte predominant Hodgkins lymphoma and classical Hodgkins lymphoma. Classical Hodgkins lymphoma can be split into four subtypes: nodular sclerosing, mixed cellularity, lymphocyte rich and lymphocyte deplete.

Lymphocyte deplete carries the worst prognosis.

181
Q

Which of the following conditions causes a wide, fixed split S2?
A) Congestive heart failure
B) LBBB
C) Moderate VSD
D) RBBB with CCF
E) WPW

A

A wide, fixed S2 is typically associated with an ASD. In this case, RBBB splits the sound and the heart failure fixes the split.

LBBB and AS are associated with paradoxical splitting of S2.

182
Q

Which of the following is the first line treatment in controlling symptoms of carcinoid syndrome?
A) Conventional chemotherapy
B) Cyproheptadine
C) Interferon-a
D) Methysergide
E) Octreotide

A

Octreotide blocks the release of 5-HT and hence reduces the symptoms of the disease if the tumour cannot be resected.

183
Q

A 21M develops dyspnoea and confusion 24hrs after fracturing his left femur. Which of the following are you most likely to see on physical examination?
A) Multiple petechiae in both axilla and skin folds of the upper body
B) Multiple vesicular lesions on the back
C) Tender red nodules on the shins
D) Target lesions on the chest
E) Palpable purpura on the buttocks only

A

The appearance of showers of petechiae in both axilla and skin folds of the upper body is characteristic of fat embolism syndrome.

184
Q

Which of the following is the biggest RF in developing hepatocellular carcioma?
A) Secondary haemochromatosis
B) Diabetes mellitus
C) Aspergillus infection
D) a1-antitrypsin deficiency
E) Hepatitis E

A

Hepatocellular carcinoma usually occurs following chronic alcohol use or chronic hepatitis (Hep E is self-limiting). Metabolic syndromes that increase the risk of developing hepatocellular carcinoma include a1-antitrypsin deficiency and hereditary haemachromatosis.

185
Q

Which of the following medications is contraindicated in a patient with VT secondary to hypokalaemia?
A) Amiodarone
B) IVF with KCl
C) Lidocaine
D) Procainamide
E) Verapamil

A

Verapamil is contraindicated in VT due to the risk of causing hypotension, VF and cardiac arrest. It can, however, be used safely in SVT.

186
Q

Which of the following is the best treatment for leptospirosis?
A) Co-trimoxazole
B) Ciprofloxacin
C) Metronidazole
D) Benzylpenicillin
E) Erythromycin

A

Benylpenicillin or doxycycline.

An LP should ideally be done first to confirm meningeal involvement.

187
Q

What is the most appropriate management for an asymptomatic male who has slept with a woman who has tested positive for Chlamydia 2 months ago?
A) Reassure symptoms would have presented by now
B) Offer antibiotic therapy
C) Offer Chlamydia testing and antibiotic treatment without waiting for the results
D) Offer Chlamydia testing and antibiotic treatment if positive
E) Notify public health

A

Offer Chlamydia testing and antibiotic treatment without waiting for the results - treatment is given on the basis of exposure to infection rather than proven infection. Treatment should be given to anyone who has been exposed to an asymptomatic patient in the last 6M.

188
Q

Which of the following medications is most likely to prevent motion sickness?
A) Cyclizine
B) Chlorpromazine
C) Metoclopramide
D) Prochlorperazine
E) Domperidone

A

Motion sickness - hyoscine > cyclizine > promethazine

189
Q

Which of the following have been associated with an increased risk of atypical stress fractures of the proximal femoral shaft?
A) Spironolactone
B) Alendronate
C) Quetiapine
D) Venlaxaine
E) Clopidogrel

A

Bisphosphonates are associated with an increased risk of atypical stress fractures.

190
Q

What is the mechanism of action of aspirin?
A) Reversible inhibition of COX-1 and COX-2
B) Reversible inhibition of COX-1
C) Irreversible inhibition of COX-1 and COX-2
D) Reversible inhibition of COX-2
E) P2Y12 receptor antagonist

A

Aspirin is a non-reversible COX-1 and COX-2 inhibitor.

190
Q

Which of the following would be classified as Stage IIIB of Hodgkins lymphoma according to the Ann-Arbour classification?
A) Nodes on both sides of the diaphragm with pruritus
B) 2 or more lymph nodes on the same side of the diaphragm with pruritus
C) Nodes on both sides of the diaphragm with night sweats
D) 2 or more lymph nodes on the same side of the diaphragm with night sweats
E) 2 or more lymph nodes on the same side of the diaphragm with no systemic symptoms

A

Ann-Arbor staging of Hodgkins lymphoma
I: single lymph node
II: 2 or more lymph nodes/regions on the same side of the diaphragm
III: nodes on both sides of the diaphragm
IV: spread beyond lymoh nodes

Each stage may be subdivided into A or B
A = no systemic symptoms other than pruritus
B) weight loss>10% in last 6M, fever>38 degrees, night sweats

191
Q

Which one of the following techniques would be most suitable to detect and quantify a viral protein?
A) PCR
B) Northern blotting
C) Western blotting
D) Southern blotting
E) Eastern blotting

A

SNOW (South-North-West)
DROP (DNA-RNA-Protein)

Southern blotting detects DNA, Northern blotting detects RNA and Western blotting detects proteins

192
Q

Which of the following conditions is most associated with onycholysis?
A) Bullous pemphigoid
B) Raynaud’s disease
C) Osteogenesis imperfecta
D) Oesophageal cancer
E) Scabies

A

Raynaud’s disease may cause onycholysis (as can any cause of impaired circulation).

193
Q

Which of the following medications is contraindicated in absence seizures?
A) Ethosuximide
B) Sodium valproate
C) Lamotrigine
D) Carbamazepine
E) Clonazepam

A

Carbamazepine is contra-indicated in absence seizures.

194
Q

Which of the following medications is likely to cause hyponatraemia?
A) Aspirin
B) Metformin
C) Amlodipine
D) Carbimazole
E) Sertraline

A

SSRI’s are associated with hyponatraemia.

195
Q

Which of the following hypersensitivity reactions predominate int h acute phase of EAA?
A) Type I
B) Type II
C) Type III
D) Type IV
E) Type V

A

EAA is thought to occur largely due to immune-complex mediated tissue damage (type III hypersensitivity).

196
Q

Which of the following SSRIs has the highest incidence of discontinuation symptoms?
A) Paroxetine
B) Citalopram
C) Escitalopram
D) Fluoxetine
E) Sertraline

A

Paroxetine has a higher incidence of discontinuation symptoms.

197
Q

What is the antidote to dabigatran?
A) Adnexate-alpha
B) Eltrombopag
C) Idaracizumab
D) Phytomendione
E) Protamine

A

Idaracizumab is a mAb that binds directly to dabigatran molecules with a greated affinity than thrombin.

Adnexate-alpha is used for the reversal of apixaban or rivaroxaban.

198
Q

Which one of the following is a contraindication to surgery for NSCLC?
A) BMI 33kgm2
B) FEV1 1.3L
C) Continuation of smoking
D) Hypercalcaemia
E) Stage T2N1

A

Contraindications to lung cancer surgery include SVC obstruction, FEV < 1.5, MALIGNANT pleural effusion, and vocal cord paralysis.

199
Q

What is the most appropriate management for an asymptomatic patient found to be in complete heart block following an inferior MI?
A) Atropine
B) Isoprenaline
C) Conservative monitoring
D) Pacemaker insertion
E) Temporary pacing

A

Complete heart block following an inferior MI is NOT an indication for pacing, unlike with an anterior MI.

Complete heart blocks are commonly seen with inferior wall MIs and usually resolve in hours-to-days after revascularization. Right coronary artery occlusion is usually implicated as it vascularises the AV nodal artery. Bradyarrhythmias and AV block following inferior STEMI are usually transient, respond well to atropine and do not require permanent or temporary pacing.

200
Q

A study looks at whether golf actually increases the risk of medial epicondylitis. Sixty people who regularly play golf are matched to sixty people who do not play golf. Thirty of the golfers had developed medial epicondylitis at some point compared to ten of the non-golfers. What is the odds ratio of developing medial epicondylitis for people who play golf?A) 0.3
B) 3
C) 5
D) 2.5
E) 3.33

A

Odds are a ratio of the number of people who incur a particular outcome to the number of people who do not incur the outcome. The odds ratio may be defined as the ratio of the odds of a particular outcome with experimental treatment and that of control.

Odds of a golfer developing medial epicondylitis = 30 / 30 = 1. If we were calculating the risk, rather than the odds that is asked for, it would be 30 / 60 = 0.5.

Odds a non-golfer developing medial epicondylitis = 10 / 50 = 0.2. Again, the risk would be 10/60 = 0.16.

The odds ratio is therefore = 1 / 0.2 = 5

201
Q

A 25M presents with a painful, swollen left knee. He returned 4 weeks ago from a holiday in Spain. There is no history of trauma and he has had no knee problems previously. O/E he has a swollen, warm left knee with a full range of movement. His ankle joints are also painful to move but there is no swelling. On the soles of both feet you notice a waxy yellow rash. What is the most likely diagnosis?
A) Rheumatoid arthritis
B) Psoriatic arthritis
C) Gout
D) Reactive arthritis
E) Gonococcal arthritis

A

The rash on the soles is keratoderma blenorrhagica. His reactive arthritis may be secondary to either gastrointestinal infection or Chlamydi

202
Q

A 67M presents with bilateral paraesthesia in the radial aspects of both hands, over the thumbs and first fingers. He also has paraesthesia in the lateral aspects of both forearms and lower limb spasticity. HBA1c 46 mmol/mol. Nerve conduction studies and EMG show evidence of denervation. Which ONE of the following diagnoses is most likely?
A) B/L carpal tunnel syndrome
B) Degenerative cervical myelopathy
C) MS
D) Syringomyelia
E) Diabetic neuropathy

A

The presentation of DCM is very variable; early symptoms are often subtle and can vary in severity, however as a progressive condition, worsening, deteriorating or new symptoms should be a warning sign.

DCM symptoms can include any combination of:
-Pain (neck, upper or lower limbs)
-Loss of motor function (loss of digital dexterity, arm or leg weakness/stiffness)
-Loss of sensory function causing numbness
-Loss of autonomic function (urinary or faecal incontinence and/or impotence)
-Hoffman’s sign: reflex test to assess for cervical myelopathy. It is performed by gently flicking one finger on a patient’s hand. A positive test results in reflex twitching of the other fingers on the same hand in response to the flick.

203
Q

Which of the following is not seen in Klinefelters syndrome?
A) Small, firm testes
B) Lack of secondary sexual characteristics
C) Infertility
D) Increased incidence of breast cancer
E) Low GnRH levels

A

Klinefelter’s syndrome is associated with karyotype 47, XXY.

-often taller than average
-lack of secondary sexual characteristics
-small, firm testes
-infertile
-gynaecomastia - increased incidence of breast cancer
-elevated gonadotrophin levels but low testosterone

204
Q

How long dos a fistula take to develop?
A) 2-4W
B) 4-6W
C) 6-8W
D) 8-10W
E) 10-12W

A

The time taken for an arteriovenous fistula to develop is 6 to 8 weeks.

205
Q

A 43F is about to start treatment with trastuzumab for metastatic breast cancer. What is the most important investigation to perform prior to initiating treatment?
A) Pulmonary function tests
B) Echo
C) LFTs
D) CXR
E) Glucose tolerance test

A

Trastuzumab (Herceptin) - cardiac toxicity is common. An echo is usually performed before starting treatment.

206
Q

Which one of the following statements regarding leptin is incorrect?
A) Is produced by the hypothalamus
B) Stimulates the release of melanocyte-stimulating hormone
C) Obese patients have higher leptin levels
D) Plays a key role in the regulation of body weight
E) High levels decrease appetite

A

Leptin is secreted by adipose tissue and acts on satiety centres in the hypothalamus and decreases appetite. More adipose tissue (e.g. in obesity) results in high leptin levels.

Leptin stimulates the release of melanocyte-stimulating hormone (MSH) and corticotrophin-releasing hormone (CRH). Low levels of leptin stimulates the release of neuropeptide Y (NPY).

207
Q

Which one of the following statements regarding leptin is incorrect?
A) Is produced by the hypothalamus
B) Stimulates the release of melanocyte-stimulating hormone
C) Obese patients have higher leptin levels
D) Plays a key role in the regulation of body weight
E) High levels decrease appetite

A

Leptin is secreted by adipose tissue and acts on satiety centres in the hypothalamus and decreases appetite. More adipose tissue (e.g. in obesity) results in high leptin levels.

Leptin stimulates the release of melanocyte-stimulating hormone (MSH) and corticotrophin-releasing hormone (CRH). Low levels of leptin stimulates the release of neuropeptide Y (NPY).

208
Q

What is the best way to assess response to treatment for Hepatitis C?
A) Viral load
B) ALT levels
C) Prothrombin time
D) Anti-HCV antibodies
E) HepC genotype

A

Viral load

209
Q

What is the best way to assess response to treatment for Hepatitis C?
A) Viral load
B) ALT levels
C) Prothrombin time
D) Anti-HCV antibodies
E) HepC genotype

A

Viral load

210
Q

What is the most common organism would you expect to find in a patient with bronchiectasis?
A) H. influenzae
B) Moraxella cattarhalis
C) Mycoplasma pneumoniae
D) Pseudomonas aeruginosa
E) Streptococcus pneumoniae

A

Bronchiectasis: most common organism = Haemophilus influenzae

211
Q

Which of the following is a possible cause of OA?
A) Menopause
B) Ageing
C) Gout
D) Obesity
E) Pyrophosphate arthropathy

A

Obesity is one of the commonest causes for the early appearance of OA.

212
Q

A 72M with a long history of indigestion presents with episodes of regurgitation that help to relieve his feelings of fullness and retrosternal pain and dysphagia to solids and liquids with no associated pattern. There has not been any associated weight loss. What is the most likely diagnosis?
A) Achalasia
B) Benign oesophageal stricture
C) Oesophageal adenocarcinoma
D) Oesophageal squamous cell carcinoma
E) Pharyngeal pouch

A

A history of dysphagia to both solids and liquids, and regurgitation that relieves feelings of fullness is typical of achalasia.

Barium swallow can be used to show a ‘birds beak’ appearance, a CT will ne needed to rule out any underlying malignancy but the definitive diagnosis is made with oesophageal manometry.

213
Q

What is the earliest changes on X-ray for a patient with ankylosing spondylitis?
A) Blurring of the upper and lower vertebral rings and the sacroiliac junction
B) Calcification if intervertebral ligaments
C) Fusion of spinal facet joints
D) Presence of syndesmophytes
E) Sclerosis of the sacroiliac joints

A

Blurring of the upper and lower vertebral rings and the sacroiliac junction is the earliest change. Fusion and sclerosis of the sacroiliac joints occurs later in the disease.

214
Q

A 59M presents 3 months following CABG for a recent MI. His main complaint is SOB on exertion and on lying flat. His chest is clear on auscultation, has a BMI of 31 with normal bloods. His vital capacity is 88% of predicted which drops to 55% on lying down. Which of the following is the most likely cause of his SOB?
A) Diaphragmatic palsy
B) Heart failure
C) Obesity
D) Pulmonary fibrosis
E) Renal artery stenosis

A

The fall in VC indicates a possible diagnosis of diaphragmatic palsy.

215
Q

Which of the following is the most appropriate treatment for Strongyloides infection?
A) Metronidazole
B) Nitazoxanide
C) Piperazine
D) Albendazle
E) Tinidazole

A

Albendazole and ivermectin are treatments of choice in Strongyloides infections. Albendazole is given 400mg BD for 3 days and repeated after 3W if needed. Treatment usually result sin clearance of worms.

Tinidazole is the treatment for giardiasis.

216
Q

Tb treatment for 12M is recommended for which of the following
A) Miliary TB
B) TB meningitis
C) Genitourinary TB
D) TB of the spine
E) TB pericarditis

A

For TB meningitis, a 12M of rifampicin and isoniazid should be given along side pyrazinamide and another drug for the first 2M. For all non-CNS disease, a 6M course of antibiotics is appropriate.

217
Q

Tb treatment for 12M is recommended for which of the following
A) Miliary TB
B) TB meningitis
C) Genitourinary TB
D) TB of the spine
E) TB pericarditis

A

For TB meningitis, a 12M of rifampicin and isoniazid should be given along side pyrazinamide and another drug for the first 2M. For all non-CNS disease, a 6M course of antibiotics is appropriate.

218
Q

What is the MOA of lorazepam?
A) GABA agonist
B) GABA reuptake inhibitor
C) GABA transaminase inhibitor
D) Glutamate blocker
E) Neuronal potassium channel blocker

A

GABA receptors have multiple binding sites for benzodiazepines and barbiturates.

219
Q

A 50F complains of pain in her right elbow. This has been present for the past four weeks and is maximal around 4-5cm distal from the lateral aspect of the elbow joint. The pain is made worse by extending the elbow and pronating the forearm. What is the most likely diagnosis?
A) Lateral epicondylitis
B) Radial tunnel syndrome
C) Cubital tunnel syndrome
D) De Quervain’s tenosiniovitis
E) Medial epicondylitis

A

Symptoms of radial tunnel syndrome are similar to lateral epicondylitis making it difficult to diagnose
however, the pain tends to be around 4-5 cm distal to the lateral epicondyle
symptoms may be worsened by extending the elbow and pronating the forearm.

220
Q

A 62M presents with a lesion over his shin. On examination shiny, painless areas of yellow skin over the shin are found with abundant telangiectasia. What is the most likely diagnosis?
A) Pretibial myxoedema
B) Necrobiosis lipoidica diabeticorum
C) Erythema nododsum
D) Pyoderma gangrenosum
E) Syphilis

A

Erythema nodosum
-symmetrical, erythematous, tender, nodules which heal without scarring
-most common causes are streptococcal infections, sarcoidosis, IBD, and drugs (penicillins, sulphonamides, OCP)

Pretibial myxoedema
-symmetrical, erythematous lesions
-shiny, orange peel skin

Pyoderma gangrenosum
-initially small red papule
later deep, red, necrotic ulcers with a violaceous border

Necrobiosis lipoidica diabeticorum
-shiny, painless areas of yellow/red skin -typically on the shin of diabetics
-often associated with telangiectasia

221
Q

A 24F presented at 15-weeks gestation with a 1-week history of erythematous, tender lesions on the pretibial aspects of both legs. The patient reported having had a “flu-like” illness with fever, a sore throat and dry cough during the same time as these lesions developing. The “flu” symptoms resolved within 10 days without medical intervention, but the lesions on her legs persisted. An initial antistreptolysin-O titre was raised. What is the most likely diagnosis in this case?
A) Erythema ab igne
B) Erythema gyratum repens
C) Erythema nodosum
D) Erythema multiforme
E) Polymorphic eruption of pregnancy

A

Erythema nodosum is a condition that causes painful red bumps under the skin on the shins. It is one of the most common forms of panniculitis that causes inflammation of the subcutaneous fat under the skin. Erythema nodosum can occur in pregnancy due to hormonal changes and infectious with streptococcal pharyngitis being a common cause as described here.

Erythema multiforme is a hypersensitivity reaction usually triggered by infections, most commonly herpes simplex virus (HSV) and can be associated with pregnancy. However, it appears as target-like patches (dark circles with purple-grey centres) which are not described here.

222
Q

A 31-year-old woman who initially presented with abdominal pain and constipation is diagnosed with irritable bowel syndrome. Which one of the following bits of dietary advice is it least suitable to give?
A) Avoid missing meals
B) Restrictions tea and coffee to 3 cups/day
C) Increase intake of fibre such as bran and wholemeal bread
D) Reduce intake fo alcohol
E) Drink at least 8 cups of fluid/day

A

Insoluble sources of fibre such as bran and wholemeal should be avoided in IBS.

223
Q

A 31-year-old woman who initially presented with abdominal pain and constipation is diagnosed with irritable bowel syndrome. Which one of the following bits of dietary advice is it least suitable to give?
A) Avoid missing meals
B) Restrictions tea and coffee to 3 cups/day
C) Increase intake of fibre such as bran and wholemeal bread
D) Reduce intake fo alcohol
E) Drink at least 8 cups of fluid/day

A

Insoluble sources of fibre such as bran and wholemeal should be avoided in IBS.

224
Q

In osteogenesis imperfecta, a mutation in which type of collagen is responsible?
A) Type 1
B) Type 2
C) Type 3
D) Type 4
E) Type 5

A

Osteogenesis imperfecta occurs due to an abnormality in type 1 collagen.

225
Q

A young female presents with hypertension. Which of the following investigation results would be most typical in fibromuscular dysplasia?
A) Echo showing L outflow obstruction
B) Serum cholesterol >7
C) US renal tracts showing asymmetric kidneys
D) MRI abdomen showing solitary adrenal mass
E) Electrolyte profile shoiwng hypokalaemia

A

A patients who is young, female, hypertension and has asymmetric kidneys with a normal urine dip should prompt consideration of fibromuscular dysplasia.

226
Q

A 22M with Israeli ancestry presents with severe abdominal pain of several hours’ duration. He has a history of similar episodes and recurrent febrile illnesses in the past, which usually last 2 or 3 days. He reports these episodes first started around the age of 13. Given the likely diagnosis, the patient is given colchicine. What is the mode of inheritance of this condition?
A) AR
B) AD
C) Mitochondrial
D) X-linked D
E) X-linked R

A

The condition in the scenario above is familial Mediterranean fever (FMF), given the recurrent episodes of abdominal pain with associated fever. Treatment with colchicine is another clue to the diagnosis. The correct answer regarding inheritance is autosomal recessive. The condition is more commonly seen in people of Mediterranean or Arab origin.

227
Q

A 34F with ascites secondary to liver cirrhosis is admitted. The serum creatinine on admission is 95 µmol/l. Ten days after admission urine output decreases significantly and his creatinine increases to 221µmol/l. Albumin is given to correct suspected hypovolaemia. What is the most appropriate further management?
A) Octreotide
B) Propranolol
C) Terlipressin
D) Acetylcysteine
E) Dopamine

A

Hepatorenal syndrome has been categorized into two types:

Type 1 HRS
Rapidly progressive
Doubling of serum creatinine to > 221 µmol/L or a halving of the creatinine clearance to less than 20 ml/min over a period of less than 2 weeks
Very poor prognosis

Type 2 HRS
Slowly progressive
Prognosis poor, but patients may live for longer

Management options:
-vasopressin analogues (terlipressin) have a growing evidence base
-volume expansion with 20% albumin
-transjugular intrahepatic portosystemic shunt

228
Q

An 8-year-old is admitted with suspected appendicitis and has a laparoscopic appendicectomy. He is given 0.45 % sodium chloride post-operatively. When reviewed by the surgical team he has developed features of a headache, confusion, and disturbance to his gait. His Na is 128mmol/. Which is the most likely diagnosis?
A) Adverse reaction to patient controlled analgesia
B) Hyperosmolar hyperglycaemic state
C) Hyponatraemic encephalopathy
D) Normal pressure hydrocephalus
E) Central pontine myelinolysis

A

Avoidance of using hypotonic (0.45%) in paediatric patients - risk of hyponatraemic encephalopathy.

229
Q

An 8-year-old is admitted with suspected appendicitis and has a laparoscopic appendicectomy. He is given 0.45 % sodium chloride post-operatively. When reviewed by the surgical team he has developed features of a headache, confusion, and disturbance to his gait. His Na is 128mmol/. Which is the most likely diagnosis?
A) Adverse reaction to patient controlled analgesia
B) Hyperosmolar hyperglycaemic state
C) Hyponatraemic encephalopathy
D) Normal pressure hydrocephalus
E) Central pontine myelinolysis

A

Avoidance of using hypotonic (0.45%) in paediatric patients - risk of hyponatraemic encephalopathy.

230
Q

A patient is currently taking sulfasalazine as a DMARD for her rheumatoid arthritis. What is the MOA for the next logical addition to her medication?
A) Anti-TNF alpha
B) Dihydrofolate reductase inhibitor
C) JAK inhibitor
D) IL-6 inhibitor
E) Anti-CD20 monoclonal antibody

A

Inhibition of dihydrofolate reductase is the mechanism of action of methotrexate, which is considered to be an ‘anchor’ drug in rheumatoid arthritis. Before progressing to trying some of the newer biologic medications, patients with rheumatoid arthritis must try at least two conventional disease-modifying therapies (i.e. sulfasalazine, methotrexate, hydroxychloroquine or leflunomide). Hydroxychloroquine and leflunomide are not listed as answers to this question. One of the two medications trialled should be methotrexate unless contraindicated.

231
Q

You review a 72M with metastatic bowel cancer who is in the terminal phase and has a syringe driver. Unfortunately he has developed intestinal obstruction and is suffering with bowel colic. What is the most appropriate drug to add to the syringe driver?
A) Metoclopramide
B) Morphine
C) Levopromazine
D) Haloperidol
E) Hyoscine butylbromide

A

Syringe drivers: respiratory secretions & bowel colic may be treated by hyoscine hydrobromide, hyoscine butylbromide, or glycopyrronium bromide.

232
Q

What is the mode of inheritance of Marfan’s disease?
A) X-linked recessive
B) Mitochondrial
C) Autosomal codominant
D) Autosomal recessive
E) Autosomal dominant

A

Marfan’s syndrome is an autosomal dominant connective tissue disorder. It is caused by a defect in the FBN1 gene on chromosome 15 that codes for the protein fibrillin-1. It affects around 1 in 3,000 people.

233
Q

Which of the following could lead to a false-negative BNP result?
A) <age of 70
B) Obesity
C) Diabetes
D) HTN
E) Female

A

Obesity can caused decreased BNP levels, leading to a false negative for heart failure screening.

234
Q

Which of the following is the most likely finding in a patient with pulmonary fibrosis?
A) Barrelled chest
B) Bilaterally reduced chest expansion
C) Focal increased vocal resonance
D) Icterus
E) Pectus excavatum

A

Pulmonary fibrosis results in decreased lung compliance. In IPF the elastin in the connective tissue is replaced by stiff, non-compliant fibrotic scar tissue which limits the degree of chest expansion.

Increased vocal resonance is an indicator of increased tissue density, for example in consolidation associated with pneumonia. It occurs due to the clearer transmission of upper airway sounds from the vocal cords through the dense tissue. Focal tissue such as this is not found in IPF.

235
Q

Which of the following is the most likely finding in a patient with pulmonary fibrosis?
A) Barrelled chest
B) Bilaterally reduced chest expansion
C) Focal increased vocal resonance
D) Icterus
E) Pectus excavatum

A

Pulmonary fibrosis results in decreased lung compliance. In IPF the elastin in the connective tissue is replaced by stiff, non-compliant fibrotic scar tissue which limits the degree of chest expansion.

Increased vocal resonance is an indicator of increased tissue density, for example in consolidation associated with pneumonia. It occurs due to the clearer transmission of upper airway sounds from the vocal cords through the dense tissue. Focal tissue such as this is not found in IPF.

236
Q

Which of the following is the most useful test to confirm the diagnosis of invasive aspergillosis?
A) Aspergillus IgE
B) Aspergillus IgG
C) Galactomannan test
D) Positive skin test for aspergillus
E) Sputum eosinophils

A

Galactomannan test is the most useful to confirm the diagnosis if invasive aspergillosis. Galactomannan is an important component of the aspergillus cell wall and can be detected in the blood stream by the ELISA test.

237
Q

What is the mode of inheritance for Haemophilia A?
A) AD
B) AR
C) X-linked D
D) X-linked R
E) Mitochondrial

A

X-linked recessive

238
Q

Which of the following is most associated with onycholysis?
A) Alopecia areata
B) Cutaneous lupus erythematosus
C) Hypoproteinaemia
D) Lichen planus
E) Thyrotoxicosis

A

It is a well-known complication of thryotoxicosis which can also lead to thyroid acropachy.

239
Q

Which of the following is most associated with onycholysis?
A) Alopecia areata
B) Cutaneous lupus erythematosus
C) Hypoproteinaemia
D) Lichen planus
E) Thyrotoxicosis

A

It is a well-known complication of thryotoxicosis which can also lead to thyroid acropachy.

240
Q

Which of the following is the best treatment option for a patient with a BP of 195/134, retinal haemorrhages on fundoscopy (but no papilloedema) and a past medical history of asthma and systemic sclerosis?
A) Atenolol
B) IV labetalol
C) IV sodium nitroprusside
D) Nifedipine oral
E) Nifedipine sublingual

A

Malignant hypertension is a very serious, rare condition characterised by very high blood pressure, B/L retinal haemorrhages and/or exudates/cotton wool spots, without the added requirement of papilloedema.

Normal management would include IV labetalol but given her history of asthma, IV sodium nitroprusside would be best management in this case.

241
Q

What is the MOA of ticagrelor?
A) Inhibit the binding of ADP to platelets
B) Glycoprotein IIb/IIIa receptor inhibition
C) Glycoprotein IIb/IIIa receptor antagonist
D) Inhibit phosphodiesterase
E) Inhibition of adenosine reuptake

A

Ticagrelor has a similar mechanism of action to clopidogrel - inhibits ADP binding to platelet receptors.

The main target of ADP receptor inhibition is the P2Y12 receptor, as it is the one which leads to sustained platelet aggregation and stabilisation of the platelet plaque.

Examples include:
Clopidogrel
Prasugrel
Ticagrelor
Ticlopidine

242
Q

What is the MOA of digoxin?
A) Blocks Ca2+ release from the sarcoplasmic reticulum
B) Blocks Na+ entry into myocytes
C) Agonist of the myocyte sodium-calcium exchanger
D) K+ channel blocker
E) Inhibits the Na+/K+ ATPase pump

A

Digoxin - inhibits the Na+/K+ ATPase pump.

243
Q

A patient with known heart failure has slight limitation of physical activity. She is comfortable at rest but ordinary activities such as walking to the local shops results in fatigue, palpitations or dyspnoea. What New York Heart Association class best describes the severity of their disease?
A) NYHA Class 0
B) NYHA Class I
C) NYHA Class II
D) NYHA Class III
E) NYHA Class IV

A

NYHA Class I
-no symptoms
-no limitation: ordinary physical exercise does -not cause undue fatigue, dyspnoea or palpitations

NYHA Class II
-mild symptoms
-slight limitation of physical activity: comfortable at rest but ordinary activity -results in fatigue, palpitations or dyspnoea

NYHA Class III
-moderate symptoms
-marked limitation of physical activity: comfortable at rest but less than ordinary activity results in symptoms

NYHA Class IV
-severe symptoms
-unable to carry out any physical activity without discomfort: symptoms of heart failure are present even at rest with increased discomfort with any physical activity

244
Q

Which one of the following is most likely to be seen in a patient with MEN type I?
A) Phaeochromocytoma
B) Insulinoma
C) Marfanoid body habitus
D) Medullary thyroid cancer
E) RET gene

A

MEN type I
-Hyperparathyroidism
-Pituitary tumour
-Insulinoma
-Most common presentation = hypercalcaemia

MEN type IIa
-Medullary thyroid cancer
-Phaeochromocytoma
-Hyperparathyroidism

MEN type IIb
-Medullary thyroid cancer
-Phaeochromocytoma
-Marfanoid body habitus
-Neuromas

245
Q

What is the most appropriate screening test for hereditary angioedema following a suspected attack?
A) Serum IgE
B) Serum C3
C) Serum tryptase
D) Serum C4
E) Serum C1-INH levels

A

Hereditary angioedema - C4 is the best screening test in-between attacks.

-C1-INH level is low during an attack.
-low C2 and C4 levels are seen, even between attacks. Serum C4 is the most reliable and widely used screening tool.

246
Q

Which of the following medications can be used to treat SLE symptoms in a patient who is 16W pregnant?
A) Azathioprine
B) Ciclosporin
C) Cyclophosphamide
D) Methotrexate
E) Mycophenolate

A

Azathioprine

Ciclosporin appears to be associated with premature delivery and low birth weight, although it does not seem to be associated with malformations, this drives its use as an alternative to azathioprine in patients who fail to gain control of their disease. Cyclophosphamide, methotrexate and mycophenolate are all contraindicated for use in pregnancy.

247
Q

Which of the following medications can be used to treat SLE symptoms in a patient who is 16W pregnant?
A) Azathioprine
B) Ciclosporin
C) Cyclophosphamide
D) Methotrexate
E) Mycophenolate

A

Azathioprine

Ciclosporin appears to be associated with premature delivery and low birth weight, although it does not seem to be associated with malformations, this drives its use as an alternative to azathioprine in patients who fail to gain control of their disease. Cyclophosphamide, methotrexate and mycophenolate are all contraindicated for use in pregnancy.

248
Q

Which one of the following antiepileptic drugs is most associated with weight gain?
A) Ethosuximide
B) Sodium valproate
C) Levetiracetam
D) Carbamazepine
E) Lamotrigine

A

Sodium valproate may cause weight gain.

249
Q

What is the first-line treatment of ITP in a patient with a platelet count of 8.10(9)?
A) IVIG
B) IV methylprednisolone
C) Observation
D) Oral prednisolone
E) Pooled platelet transfusion

A

Generally, asymptomatic patients do not require active treatment. However, where the platelet count falls below 30* 109/L treatment is usually indicated due to the increased risk of bleeding once the platelet count falls to this level.

-Emergency treatment: life-threatening or organ threatening bleeding –> Platelet transfusion, IV methylprednisolone and IVIG
-Platelet count >30109/L –> Observation
-Platelet count <30
109/L –> PO prednisolone

250
Q

A 71F with dry age-related macular degeneration is reviewed. Unfortunately her eyesight has deteriorated over the past six months. She has never smoked and is taking antioxidant supplements. What is the most appropriate next step?
A) Retinal transplant
B) Intravitreal ranibizumab
C) Explain no other medical therapies are available
D) Photodynamic therapy
E) Photocoagulation

A

ARMD is the most common cause of blindness in the UK. Degeneration of the central retina (macula) is the key feature with changes usually bilateral. It is characterised by degeneration of retinal photoreceptors that results in the formation of drusen which can be seen on fundoscopy and retinal photography.

Explain no other medical therapies are available - the mainstay of treatment for dry ARMD is a combination of zinc with anti-oxidant vitamins A, C and E. For wet ARMD however, anti-VEGF agents may limit progression.

251
Q

Which of the following medications is contra-indicated in G6PD deficiency?
A) Amikacin
B) Ciprofloxacin
C) Clarithromycin
D) Doxycycline
e) Tobramycin

A

Ciprofloxacin is contraindicated in G6PD deficiency as such patients could experience potentially fatal haemolytic anaemia.

252
Q

A patient with glaucoma and T2DM is on aetazolamide, metformin and atenolol. She is asymptomatic but has abnormal blood results: Na 137, K 3.5, Cl 115, bicarb 15. Her pH is 7.26 and pCO2 is 2.9 kPa. Which of the following is the most likely cause of her acid-base disturbance?
A) Inhibition of carbonic anhydrase in the proximal tubule
B) Ketoacidosis
C) Metformin induced lactic acidosis
D) Stress induced hyperventilation
E) Type IV renal tubular acidois

A

Acetazolamide therapy commonly chronic asymptomatic metabolic acidosis with a normal anion gap and partial respiratory compensation through inhibition of carbonic anhydrase in the proximal tubule.

Ketoacidosis and lactic acidosis are both prominent causes of raised anion-gap acidosis.

253
Q

How would the power of a study be correctly defined?
A) The probability of not rejecting the null hypothesis when it is false
B) The probability of not rejecting the null hypothesis when it is true
C) The probability detecting a statistically significant result
D) The probability of rejecting the null hypothesis when it is false
E) The probability of rejecting the null hypothesis when it is false

A
254
Q

How would the power of a study be correctly defined?
A) The probability of not rejecting the null hypothesis when it is false
B) The probability of not rejecting the null hypothesis when it is true
C) The probability detecting a statistically significant result
D) The probability of rejecting the null hypothesis when it is false
E) The probability of rejecting the null hypothesis when it is true

A

The power of the study is the probability of correctly rejecting the null hypothesis when it is false.

A significant p-value (p<0.05) may lead to rejection of the null hypothesis when it is actually true - this is known as a type I error. The p-value may be non-significant (>0.05) when the null hypothesis is false - this is known as a type II error.

255
Q

Where in the cell is circular, double-stranded DNA found?
A) Golgi body
B) Mitochondria
C) Nucleolus
D) Nucleus
E) Ribosomes

A

Mitochondria

255
Q

Where in the cell is circular, double-stranded DNA found?
A) Golgi body
B) Mitochondria
C) Nucleolus
D) Nucleus
E) Ribosomes

A

Mitochondria

256
Q

Which of the following is likely to be most effective in reversing the effects of alteplase?
A) FFP
B) Protamine sulphate
C) Prothrombin complex concentrate
D) Tranexamic acid
E) Vitamin K

A

Tranexamic acid is a potent inhibitor of the activation of plasminogen to plasmin. At higher concentrations it is a non-competitive inhibitor of plasmin.

257
Q

A patient is uncontrolled on hydrocortisone 1% cream for her atopic eczema. What is the next step in management?
A) Betamethasone valerate (0.1%)
B) Clobetasone butyrate (0.05%)
C) Clobetasol proprionate (0.05%)
D) Topical tetracycline
E) Regular wet wraps

A

Moderate: Clobetasone butyrate 0.05%
Potent: Betamethasone valerate 0.1%
Very potent: Clobetasol propionate 0.05%

258
Q

Which of the following medications is most likely to precipitate acute angle closure glaucoma?
A) Aspirin
B) Propranolol
C) Sodium valproate
D) Sumatriptan
E) Topiramate

A

Topiramate is used as a first-line agent in the prophylaxis of migraine. It is an important cause of drug-induced acute angle closure glaucoma, typically occurring within one month of treatment. Topiramate-induced acute angle closure glaucoma is usually treated with cycloplegia and topical steroids alone rather than laser peripheral iridotomy.

259
Q

What is the most appropriate treatment of anthrax?
A) Aciclovir
B) Ciprofloxacin
C) Flucloxacillin
D) Fluconazole
E) Surgical debridement

A

Anthrax is caused by Bacillus anthracis, a Gram positive rod. It is spread by infected carcasses. The recommended initial treatment is ciprofloxacin.

260
Q

A 61M is treated for ACS. A few days later a diagnostic coronary angiogram is performed. The following week a deteriorating of renal function is noted associated with a purpuric rash on his feet. What is the most likely diagnosis?
A) Aspirin induced interstitial nephritis
B) HIT
C) Renal artery stenosis
D) Cholesterol embolisation
E) Antiphospholipid syndrome

A

Cholesterol embolisation is a well-documented complication of coronary angiography.

eosinophilia
purpura
renal failure
livedo reticularis

261
Q

A 61M is treated for ACS. A few days later a diagnostic coronary angiogram is performed. The following week a deteriorating of renal function is noted associated with a purpuric rash on his feet. What is the most likely diagnosis?
A) Aspirin induced interstitial nephritis
B) HIT
C) Renal artery stenosis
D) Cholesterol embolisation
E) Antiphospholipid syndrome

A

Cholesterol embolisation is a well-documented complication of coronary angiography.

eosinophilia
purpura
renal failure
livedo reticularis

262
Q

What does Troponin T bind to when measured as a cardiac biomarker?
A) Actin
B) Calcium ions
C) Myoglobin
D) Tropomyosin
E) Troponin I

A

This biomarker binds to tropomyosin forming the troponin-tropomyosin complex which regulates muscle contraction. As with the other troponin subunits (troponin I and troponin C), troponin T is released in cardiac cell damage (e.g. in acute coronary syndrome) although it is considered less sensitive and specific than troponin I in the diagnosis of myocardial infarction.

Actin is bound to Troponin I. Calcium ions are bound to by Troponin C.

262
Q

What does Troponin T bind to when measured as a cardiac biomarker?
A) Actin
B) Calcium ions
C) Myoglobin
D) Tropomyosin
E) Troponin I

A

This biomarker binds to tropomyosin forming the troponin-tropomyosin complex which regulates muscle contraction. As with the other troponin subunits (troponin I and troponin C), troponin T is released in cardiac cell damage (e.g. in acute coronary syndrome) although it is considered less sensitive and specific than troponin I in the diagnosis of myocardial infarction.

Actin is bound to Troponin I. Calcium ions are bound to by Troponin C.

263
Q

How long will you monitor a patient treated with two doses of adrenaline for anaphylaxis before considering discharge?
A) 1 hour
B) 2 hours
C) 6 hours
D) 48 hours
E) 1 week

A

A biphasic reaction includes a recurrence of symptoms that develops after the apparent resolution of the initial reaction. Biphasic reactions have been reported to occur in 1%-20% of anaphylaxis episodes.

Resus Council UK guidelines recommend observing the patient for at least 6 hours if a second dose of adrenaline is given. Patients may be discharged after 2 hours if they have responded to one dose of IM adrenaline. If they require more than 2 doses, then patients will need to stay for 12 hours following symptom resolution.

264
Q

A 40M presents with a productive cough that has been worsening for the past six months. He is a known IVDU and has had multiple episodes of pneumonia in the past. He has conjunctival pallor and bilateral wheezing. Laboratory studies show: Eosinophils - 7 * 109/L (0 - 0.4* 109/L)
Serum IgE - 1000 kU/L (5 - 120 kU/L)
Sputum microscopy - eosinophils and fungal hyphae
CXR shows proximal bronchiectasis and consolidations in the right upper lobe.
Which of the following is the treatment of choice for this patient?
A) Nebulised albuterol
B) Inhaled beclometasone
C) PO prednisolone
D) PO itraconazole
E) IV amphotericin

A

The history here is in keeping with a history of allergic bronchopulmonary aspergillosis.

Oral glucocorticoids is the treatment of choice in allergic bronchopulmonary aspergillosis. High dose prednisolone is used for acute management followed by low to medium dosing for maintenance. It is then tapered off over 3-12 months. Prednisolone decreases the risk of bronchiectasis.

Intravenous amphotericin is one of the medications that can be used in invasive aspergillosis.

265
Q

A 65F presents with 3 months of weight loss and nausea. Over the past 2-weeks, she has noticed yellowing of her skin and she has an all-over body itch. She describes fatty, difficult-to-flush stools. Examination reveals a non-tender palpable gallbladder. Bloods show a raised ALP and bilirubin. What investigation should be performed to confirm the diagnosis?
A) Abdominal USS
B) ERCP
C) High resolution CT pancreas
D) MRCP
E) PET CT and pancreas

A

The patient’s history (weight loss, jaundice, pruritis and steatorrhea), examination (palpable gallbladder - Courvoisier’s sign) and blood tests (obstructive jaundice) are most suggestive of pancreatic cancer. High resolution CT is the diagnostic investigation of choice.

265
Q

A 65F presents with 3 months of weight loss and nausea. Over the past 2-weeks, she has noticed yellowing of her skin and she has an all-over body itch. She describes fatty, difficult-to-flush stools. Examination reveals a non-tender palpable gallbladder. Bloods show a raised ALP and bilirubin. What investigation should be performed to confirm the diagnosis?
A) Abdominal USS
B) ERCP
C) High resolution CT pancreas
D) MRCP
E) PET CT and pancreas

A

The patient’s history (weight loss, jaundice, pruritis and steatorrhea), examination (palpable gallbladder - Courvoisier’s sign) and blood tests (obstructive jaundice) are most suggestive of pancreatic cancer. High resolution CT is the diagnostic investigation of choice.

266
Q

Which one of the following conditions is most associated with a bisferiens pulse?
A) Cardiac tamponade
B) Severe LVF
C) Aortic stenosis
D) PDA
E) Mixed aortic valve disease

A

Pulsus paradoxus
greater than the normal (10 mmHg) fall in systolic blood pressure during inspiration → faint or absent pulse in inspiration
severe asthma, cardiac tamponade

Slow-rising/plateau
aortic stenosis

Collapsing
aortic regurgitation
patent ductus arteriosus
hyperkinetic states (anaemia, thyrotoxic, fever, exercise/pregnancy)

Pulsus alternans
regular alternation of the force of the arterial pulse
severe LVF

Bisferiens pulse
‘double pulse’ - two systolic peaks
mixed aortic valve disease

‘Jerky’ pulse
hypertrophic obstructive cardiomyopathy*

267
Q

A middle-aged man is diagnosed with nasopharyngeal carcinoma. What type of virus family is associated with this malignancy?
A) Reovirus
B) Herpesvirus
C) Parvovirus
D) Adenovirus
E) Hepadnaviridae

A

The Epstein-Barr virus is one of the herpes viruses.

Malignancies associated with EBV infection
Burkitt’s lymphoma*
Hodgkin’s lymphoma
nasopharyngeal carcinoma
HIV-associated central nervous system lymphomas

268
Q

What Gell and Coombs hypersensitivity class is Myasthenia Gravis?
A) Type I
B) Type II
C) Type III
D) Type IV
E) Type V

A

Type - Antibodies that recognise and bind to the cell surface receptors.

This either stimulating them or blocking ligand binding * Graves’ disease
* Myasthenia gravis

269
Q

What is the mechanism of action of DDAVP in von Willebrand’s disease?
A) Prevents renal excretion of vWF
B) Inhibits breakdown of vWF
C) Induces release of Factor VIII
D) Induces release of vWF
E) Acts as a substitute carrier for vWF

A

Desmopressiin - induces release of von Willebrand’s factor from endothelial cells.

270
Q

What is the mechanism of action of DDAVP in von Willebrand’s disease?
A) Prevents renal excretion of vWF
B) Inhibits breakdown of vWF
C) Induces release of Factor VIII
D) Induces release of vWF
E) Acts as a substitute carrier for vWF

A

Desmopressiin - induces release of von Willebrand’s factor from endothelial cells.

271
Q

A 44M presents with a 2W history of visual and hearing loss. He is otherwise well. He has no past medical history. On examination, you note bilateral central scotomas and sensorineural hearing loss. Bloods show a metabolic acidosis with a normla anion gap, hypokalaemia and a raised glucose. What is the most likely diagnosis?
A) Alports syndrome
B) Lead poisoning
C) Mercury poisoning
D) Methanol poisoning
E) MS

A

The presence of hypokalemia with NAGMA is suggestive of type 1 RTA.

The presence of visual field defects, hearing loss and RTA favours the diagnosis of mercury poisoning.

272
Q

What treatment would be advised for a female hoping to get pregnant who has Graves disease?
A) Beta-blockers alone
B) 12-18M thionamide therapy
C) A combination of anti-thyroid drugs and thyroxine
D) Radioactive iodine
E) Thyroidectomy

A

Propylthiouracil therapy should be used in females hoping to become pregnant and in their first trimester of pregnancy with close monitoring or LFTs.

B-blockers should not be given in pregnancy of lactation unless absolutely essential due to increased risk of reduced placental perfusion and hence increased risk of inta-uterine death.

273
Q

Which of the following medications is most likely to cause acute pancreatitis?
A) Aspirin
B) Azathioprine
C) Budesonide
D) Paracetamol
E) Sulfasalazine

A

Azathioprine is associated with a number of significant side effects:
-Pancreatitis
-Hypersensitivity reactions
-Interstitial nephritis
-Liver disease

274
Q

What is the best treatment for a pregnant patient with asymptomatic bacteriuria at booking?
A) 7d amoxicillin
B) 7d levofloxacin
C) Repeat sample
D) No action
E) 7d nitrofurantoin

A

7d nitrofurantoin - amoxicillin should only be used when the bacteria are known to be sensitive to it.

275
Q

Which of the following are most likely to be found in a patient with minimal change nephropathy?
A) Low C3 complement concentration
B) Normal HDL levels
C) Presence of monoclonal band on electrophoresis
D) Raised anti-streptolysin O titre
E) Reduced a- and b- fractions

A

Nephrotic syndrome = proteinuria, hypoalbuminaemia and peripheral oedema. It is associated with raised serum cholesterol levels but it is LDL levels that are raised; HDL is normal.

276
Q

Which of the following are most likely to be found in a patient with minimal change nephropathy?
A) Low C3 complement concentration
B) Normal HDL levels
C) Presence of monoclonal band on electrophoresis
D) Raised anti-streptolysin O titre
E) Reduced a- and b- fractions

A

Nephrotic syndrome = proteinuria, hypoalbuminaemia and peripheral oedema. It is associated with raised serum cholesterol levels but it is LDL levels that are raised; HDL is normal.

277
Q

Which of the following complement factor deficiencies is most associated with early onset SLE?
A) C3
B) C4
C) C5
D) C6
E) C7

A

C4 - deficiencies of early parts of the classical pathway are associated with early onset SLE. SLE is MOST associated with C1q deficiency but also in those with C4 deficiency.

278
Q

A 45F presents with erythroderma on a history of chronic skin problems that have required long-standing dermatology treatment. What is the most likely underlying cause of her presentation?
A) Drug eruption
B) Eczema
C) Mycosis fungoides
D) Pityriasis rubra pilaris
E) Underlying malignancy

A

Erythroderma is the term applied to any inflammatory skin disorder covering >90% of the body which, in adults, is most likely due to eczema and psoriasis.

279
Q

Which physiological change is most likely to have occurred in an elderly patient that will increase their risk of heat stroke?
A) Decreased peripheral dilatation
B) Decreased sweating
C) Increased basal metabolic rate
D) Increased peripheral dilatation
E) Increased sweating

A
280
Q

Which physiological change is most likely to have occurred in an elderly patient that will increase their risk of heat stroke?
A) Decreased peripheral dilatation
B) Decreased sweating
C) Increased basal metabolic rate
D) Increased peripheral dilatation
E) Increased sweating

A

The elderly lose the ability to sweat effectively which increases their risk of heat stroke.

281
Q

Which phase of clinical trials specifically looks at the efficacy of the drug?
A) Phase I
B) Phase IIa
C) Phase IIb
D) Phase III
E) Phase IV

A

Phase I - Determines pharmacokinetics and pharmacodynamics and side-effects prior to larger studies - Conducted on healthy volunteers

Phase II - Assess efficacy + dosage - Involves small number of patients affected by particular disease

May be subdivided into
IIa - assesses optimal dosing
IIb - assesses efficacy

Phase III - Assess effectiveness - Typically involves 100-1000’s of people, often as part of a randomised controlled trial, comparing new treatment with established treatments

Phase IV - Postmarketing surveillance - Monitors for long-term effectiveness and side-effects

282
Q

What would be the appropiate dose if you were switching a patient from 30mg MST BD to be put through a syringe driver?
A) 60mg over 24hrs
B) 30mg over 24hrs
C) 40mg over 24hrs
D) 10mg over 24hrs
E) 6mg over 24hrs

A

30mg over 24hrs

Calculate the total dose over 24hrs
Divide by 2 if converting PO to S/C

283
Q

Which of the following agents would be most likely to reduce bleeding in a patient recently started on dabigatran?
A) PCC
B) Tranexamic acid
C) 72hr omeprazole infusion
D) Haemodialysis
E) Idaracizumab (praxbind)

A

Darucizumab is a recently developed monoclonal antibody fragment which binds dabigatran with an affinity that is 350 times as high as with thrombin. Consequently, idarucizumab binds free and thrombin-bound dabigatran and rapidly neutralises its activity. Currently, dabigatran is the only direct acting oral anticoagulant to have a commercially available antidote, although others are in development.

284
Q

A 45F with a history of primary Sjogren’s syndrome is reviewed in clinic. Her main problem is a dry mouth, which unfortunately has not responded to artificial saliva. Which one of the following medications is most likely to be beneficial?
A) Rivastigmine
B) Neostigmine
C) Clonidine
D) Atropine
E) Pilocarpine

A

Management
-artificial saliva and tears
-pilocarpine may stimulate saliva production

285
Q

Which of the following may differentiate CML from a leukaemia reaction in a patient with an elevated WCC following sepsis?
A) Left shift neutrophils
B) Leucocytosis <50x10(9)
C) Low leucocyte alkaline phosphatase score
D) Presence of promyelocytes
E) Toxic granulation in WCC

A

Differentiating chronic myeloid leukaemia from leukaemoid reactions: leukocyte alkaline phosphatase score is low in CML, high in leukaemoid reaction.

A leukaemoid reaction is the presence of immature cells in the peripheral blood and can result from severe infection, haemolysis, haemorrhage or metastatic cancer with BM infiltration. Leukemoid reactions in themselves are benign (although normal indication severe disease) however it is important to exclude CML which can present with identical findings on a peripheral blood film. Leukocyte alkaline phosphatase is found within mature white blood cells and therefore can be used to assist in differentiating between a leukaemoid reaction and CML. Lower levels (i.e. a low leukocyte alkaline phosphatase score) are found in conditions involving undeveloped leukocytes such as in CML, whereas higher levels are seen in the physiological response (i.e. a leukaemoid reaction).

286
Q

A 58F attends the clinic with a 6M history of rectal bleeding and pain. Her consultant decides to arrange a proctoscopy.
Macroscopic - Erythematous ulcerated plaque in close proximity to the pectinate line
Biopsy - Results suggestive of squamous cell carcinoma
Given the diagnosis, which of the following is the strongest risk factor?
A) HIV infection
B) HPV infection
C) Immunosuppressant drugs
D) Smoking
E) Past history of cervical cancer

A

HPV infection is the strongest risk factor for anal cancer. HPV infection causes 80-85% of SSCs of the anus (usually HPV16 or HPV18 subtypes).

287
Q

A 56F is admitted to ITU with a severe pneumonia. Thyroid function tests are most likely to show:
A) Normal TSH, high T4, high T3
B) Normal/low TSH, low T4, low T3
C) High TSH, low T4, low T3
D) Low TSH, high T4, high T3
E) High TSH, normal T4, high T3

A

In sick euthyroid syndrome (now referred to as non-thyroidal illness) it is often said that everything (TSH, thyroxine and T3) is low. In the majority of cases however the TSH level is within the >normal range (inappropriately normal given the low thyroxine and T3).

288
Q

A 41M presents with a persistent cough. This has been present for 6M and has been coughing up blood for the past 2W. He feels more short-of-breath when exerting himself than normal. He is a non-smoker and has no past medical history. He has reduced AE in the RUZ. CXR shows a RUL collapse and a subsequent bronchoscopy demonstrates a cherry-red lesion in the right superior lobar bronchus. What is the most likely diagnosis?
A) Small cell lung carcinoma
B) Large cell lung carcinoma
C) Lung carcinoid
D) Broncheoalveolar carcinoma
E) Bronchial adenocarcinoma

A

The ‘cherry-red’ lesion is a typical finding of lung carcinoid.

typical age = 40-50 years
smoking not risk factor
slow growing: e.g. long history of cough, recurrent haemoptysis
often centrally located and not seen on CXR
‘cherry red ball’ often seen on bronchoscopy
carcinoid syndrome itself is rare (associated with liver metastases)

289
Q

Which cell surface protein does the Epstein-Barr virus bind to?
A) CD3
B) CD4
C) CD24
D) CD15
E) CD21

A

CD21 is the receptor for the Epstein-Barr virus.

290
Q

A 52F who has primary atrophic hypothyroidism is reviewed following recent thyroid function tests (TFTs):
TSH 12.5 mU/l
Free T4 14 pmol/l
She is currently taking 75mcg of levothyroxine once a day. How should these results be interpreted?
A) Poor compliance with medication
B) Her thryoxine dose needs to be increased
C) Evidence of recent systemic therapy
D) She is on the correct dose
E) T4-T3 conversion disorder

A

The TSH level is high. This implies that over recent days/weeks her body is thyroxine deficient. However, her free T4 is within normal range. The most likely explanation is that she started taking the thyroxine properly just before the blood test. This would correct the thyroxine level but the TSH takes longer to normalise.

291
Q

Which one of the following is the most common ocular manifestation of rheumatoid arthritis?
A) Scleritis
B) Episcleritis
C) Keratoconjunctivitis sicca
D) Corneal ulceration
E) Keratitis

A

Keratoconjunctivitis sicca is characterised by dry, burning and gritty eyes caused by decreased tear production.

292
Q

A 62M presents with severe pain affecting his R shoulder worst in the middle range of abduction, He is unable to initiate abduction via active movement, although passive movement is less painful. XR is normal. What diagnosis fits best with this clinical picture?
A) Acromio-clavicular OA
B) Adhesive capsulitis
C) Pseudogout
D) Supraspinatous tendonitis
E) The Milwaukee shoulder

A

Supraspinatous tendonitis is one of the commonest causes of painful restriction of shoulder movement across all ages. Pain radiates to the upper arm and is made worse by active abduction and elevation. Passive movement releases the impingement of the tendon and is often less painful. Supraspinatous muscle is the main one involved in the initial range of abduction - particularly the first 15 degrees, after which the deltoid muscle predominate. Impaired initiation of abduction is typically present.

Management = NSAIDS or local corticosteroid injections.

Adhesive capsulitis = pain and stiffness across all motions of the joint on both active and passive movement

292
Q

A 62M presents with severe pain affecting his R shoulder worst in the middle range of abduction, He is unable to initiate abduction via active movement, although passive movement is less painful. XR is normal. What diagnosis fits best with this clinical picture?
A) Acromio-clavicular OA
B) Adhesive capsulitis
C) Pseudogout
D) Supraspinatous tendonitis
E) The Milwaukee shoulder

A

Supraspinatous tendonitis is one of the commonest causes of painful restriction of shoulder movement across all ages. Pain radiates to the upper arm and is made worse by active abduction and elevation. Passive movement releases the impingement of the tendon and is often less painful. Supraspinatous muscle is the main one involved in the initial range of abduction - particularly the first 15 degrees, after which the deltoid muscle predominate. Impaired initiation of abduction is typically present.

Management = NSAIDS or local corticosteroid injections.

Adhesive capsulitis = pain and stiffness across all motions of the joint on both active and passive movement

293
Q

A 62M presents with severe pain affecting his R shoulder worst in the middle range of abduction, He is unable to initiate abduction via active movement, although passive movement is less painful. XR is normal. What diagnosis fits best with this clinical picture?
A) Acromio-clavicular OA
B) Adhesive capsulitis
C) Pseudogout
D) Supraspinatous tendonitis
E) The Milwaukee shoulder

A

Supraspinatous tendonitis is one of the commonest causes of painful restriction of shoulder movement across all ages. Pain radiates to the upper arm and is made worse by active abduction and elevation. Passive movement releases the impingement of the tendon and is often less painful. Supraspinatous muscle is the main one involved in the initial range of abduction - particularly the first 15 degrees, after which the deltoid muscle predominate. Impaired initiation of abduction is typically present.

Management = NSAIDS or local corticosteroid injections.

Adhesive capsulitis = pain and stiffness across all motions of the joint on both active and passive movement

294
Q

A 62M presents with severe pain affecting his R shoulder worst in the middle range of abduction, He is unable to initiate abduction via active movement, although passive movement is less painful. XR is normal. What diagnosis fits best with this clinical picture?
A) Acromio-clavicular OA
B) Adhesive capsulitis
C) Pseudogout
D) Supraspinatous tendonitis
E) The Milwaukee shoulder

A

Supraspinatous tendonitis is one of the commonest causes of painful restriction of shoulder movement across all ages. Pain radiates to the upper arm and is made worse by active abduction and elevation. Passive movement releases the impingement of the tendon and is often less painful. Supraspinatous muscle is the main one involved in the initial range of abduction - particularly the first 15 degrees, after which the deltoid muscle predominate. Impaired initiation of abduction is typically present.

Management = NSAIDS or local corticosteroid injections.

Adhesive capsulitis = pain and stiffness across all motions of the joint on both active and passive movement

295
Q

A nurse has a positive tuberculin skin test following exposure to patient with pulmonary TB. She has never received the BG vaccine. Her CXR and bloods are normal. She ha started a course of rifampicin and isoniazid What is the most appropriate occupation health advice?
A) Continue isoniazid and rifampicin for at least 3M
B) Continue isoniazid and rifampicin for at least 1M and isoniazid for a further 2M
C) Stay off work and repeat CXR in 3W
D) Stay off work whilst on initial prophylactic isoniazid dose for 2W
E) Stay off work for 6W

A

Healthcare workers with a positive tuberculin skin test after exposure to TB should be treated with 3M of isoniazid and rifampicin or isoniazid alone for 6M. The positive tuberculin skin test raises the possibility of latent TB. Given there are no signs of active Tb, there is no reason to stay off work.